Tag Archives: edmond hamilton

[April 10, 1969] Low (May 1969 Amazing)


by John Boston

Here’s the May Amazing, the latest installment of the dreary soap opera that this magazine has become.  The well-qualified Ted White is the new editor, the fourth in ten issues.  Though he’s listed as Managing Editor, and Sol Cohen as Editor and Publisher, White’s editorial makes it clear that he will be running the magazine—within the constraints of Cohen’s policies, of course, most notably the reprint policy.


by Johnny Bruck

As a debut issue, this one does not impress, but that’s probably not a fair judgment.  Given the abrupt departure of White’s predecessor Barry Malzberg, it was likely a scramble to get any issue at all together from available parts.  The fiction contents include an Edmond Hamilton story in a series that has run in Amazing and Fantastic for several years, publication no doubt foreordained; one very short new story; and the usual heavy load of reprints, all from the 1950s consistent with recent practice.  The non-fiction includes, as usual of late, a Laurence Janifer movie review (Barbarella—he likes it!) and a Leon Stover “Science of Man” article.  The only identifiable change is a letter column.  The book review department is missing, one hopes temporarily, since it has been one of the magazine’s brighter aspects.

As for future plans, White provides a rather carefully argued editorial, which starts by analogizing the “New Thing” in science fiction to the ongoing innovations in popular music, noting that despite the “sudden flowering” of rock music, it isn’t forgetting its roots.  After some commentary on the New Thing, sympathetic but cautionary (“One J.G. Ballard can be important, but ten little Ballards?”), White asserts that most of the “New Wave” writers have not neglected their predecessors, citing Zelazny and Delany, noting particularly that Delany has absorbed and transformed old Planet Stories-style space opera plots. “It is my conviction that the science fiction field needs a magazine in which the old and the new can exist side by side, each thriving from its proximity to the other.  And that is what I intend for Amazing: Something of the old (the reprints) and of the new (the best of the new writers). . . .” And he concludes by adding that this issue’s “Star Kings” novelet by Edmond Hamilton exemplifies exploration of the genre’s roots—but next issue we can expect a “new and very different novel by Robert Silverberg.”

It’s all gracefully done, touching the necessary bases with plausible conviction, and starkly contrasting with Harry Harrison’s pandering editorial of February 1968, which made essentially the same substantive points but which struck me as “a disappointingly smarmy exercise in having it both ways.”

The letter column is divided among sober commentary on current SF, the pleasures of letter columns and fanzine reviews, and a quite long letter contesting Stover’s “Science of Man” article War and Peace, which White says he cut down from 14 pages.  Shades of Brass Tacks!  This feature will require some tightening up but White clearly takes it seriously.  As for the reference to fanzine reviews, White promises “fan features” in both Amazing and Fantastic.

And up front—though looking backward—is another cliched cover illustration by Johnny Bruck.  Last issue, fellow Journeyer Cora Buhlert wished that Amazing would use the good Bruck covers rather than the dull ones.  Yes!  If there are any.

The Horror from the Magellanic, by Edmond Hamilton

The lead story is Edmond Hamilton’s “short novel” (33 pages), The Horror from the Magellanic, latest in his series of sequels to his 1947 novel The Star Kings.  I won’t repeat my previous jaundiced comments on the whole enterprise, but will leave it at a couple of samples:

“ ‘Highness, they’ve come out of the Marches.  The Counts’ fleet.  They’re more than twice as strong as we expected . . . and they’re coming full speed toward Fomalhaut!’
“Chapter Two
“Gordon felt a chilling dismay.  The Counts of the Marches were throwing everything they had into this.  And whether their gamble succeeded or not, in the dark background brooded the unguessable purposes and menace of the H’harn.”

And:

“. . . Gordon sat for a long time looking past the moving lights and the uproar and clamorous confusion of the great city, toward the starry sky.  A star kingdom might fall, Narath might realize his ambition and sit on the throne of Fomalhaut, and he, John Gordon, and Lianna might be sent to their deaths.  And that would be a world tragedy as well as tragedy for them.
“But if the H’harn succeeded, that would be tragedy for the whole galaxy, a catastrophe of cosmic dimension.  Thousands of years before they had come from the outer void, bent on conquest, and only the power of the Disruptor, unloosed by Brenn Bir, had driven them back .  Out there in the Lesser Magellanic Cloud they had brooded all this time, never giving up their purpose, filtering back gradually in secret plotting with the Counts, plotting with Narath, making ready some new tremendous stroke.
“Doomsday had come again, after those thousands of years.”


by Dan Adkins

To my taste, this is all an idea whose time has passed.  No disrespect to Hamilton—a working professional writing in a mode he virtually invented—especially since he has shown he can work quite capably in styles other than this bombastic costume drama (see his 1960 novel The Haunted Stars).  Three stars, acknowledging the craft involved, even if I can’t get interested.

Yesterdays, by Ray Russell

The new short story (very short), Ray Russell’s Yesterdays, couples two ancient themes, time running backwards and mad scientists; it’s clever and facile, as one would expect from the long-time fiction editor of Playboy, but no more. Three stars.

The Invaders, by Murray Leinster

The longest story in the issue is Murray Leinster’s The Invaders, from the April/May 1953 issue of Amazing, the first in its short-lived experiment in paying more in order to get better material from more well-known authors.  Leinster shared the contents page with Heinlein, Sturgeon, and Bradbury.  Unfortunately his story begins well but undermines itself, unusually for this professional of decades’ standing.


Uncredited

The scene is set in terms of purest Cold War paranoia.  The protagonist, surveying in Greece, flees an unacknowledged incursion by Bulgarian soldiers, and the author observes:

“It was not the time for full-scale war.  Bulgaria and the other countries in its satellite status were under orders to put a strain upon the outside world.  They were building up border incidents and turmoil for the benefit of their masters.  Turkey was on a war footing, after a number of incidents like this.  Indo-China was at war.  Korea was an old story.  Now Greece.  It always takes more men to guard against criminal actions than to commit them. . .  This was cold war.”

In the midst of this covert crisis, the protagonist discovers powerful evidence of infiltration by extraterrestrials in human guise—but what to do?  Who will believe him?  Leinster builds an atmosphere of suspense and suspicion at first, but it is quickly dissipated by hints that something different and more benign is going on, and by the end there’s no suspense or surprise.  Three stars, barely; it’s at least slickly readable, as usual for Leinster.

King of the Black Sunrise, by Milton Lesser

Milton Lesser’s King of the Black Sunrise is an entirely more rancid kettle of fish.  It’s from Amazing, May 1955, in the midst of the Howard Browne/Paul Fairman era of calculated formulaic mediocrity, and shows it.  It reads like the result of a barroom bet over how many egregious cliches the author could cram into a single story. 

Kent Taggert, fugitive from justice on murder charges (but of course he’s innocent), is tracked down on the obscure planet Argiv by a woman who wants to hire him for a dangerous assignment.  “I looked at her for the first time.  She was beautiful.  So damned beautiful and so damned sure of herself.  I felt like poking her one.” A bit later: “I could smell her perfume, not the kind that slams two sexy fists into your nostrils but the subtle kind, like the girls can buy only on Earth.”


Uncredited

The woman (named Helen, we later learn) discloses that the World Bureau of Investigation is on his trail, and like clockwork, a guy “who was trying too hard not to look like law” shows up at the bar where this conversation is occurring.  Taggert decides he’d better take Helen’s proposition—to guide her party to find and plunder the treasure of the Black Sunrise. 

See, Argiv has three suns—per the natives, the Green God, the Yellow God, and (“greatest of all”) the Purple God.  They all rise and set at different times, but occasionally they are all below the horizon at the same time.  That’s the Black Sunrise, even though it’s really a sunset.  During the Black Sunrise, the barrier to the natives’ treasure cave opens up, and new offerings are deposited to make sure the three Gods come back.  No one who has sought to steal this treasure has emerged alive.

So our freebooters hire some native bearers (“big flabby purple-skinned Argivians”) and march into the jungle (“King Solomon’s Mines, a hundred parsecs out in deep space,” muses Cotton, the hotheaded jerk of the party).  But soon enough the bearers become fearful and desert, and the humans must push on without much of their equipment.

It goes on in similar vein, but recounting it is even more tedious than reading it.  One star.

Wish It Away, by Frank Freeman

Frank Freeman’s Wish It Away (Fantastic, January-February 1954) is a jokey vignette so inane it almost hurts to describe it.  Protagonist Mervin sees a monster every night, psychiatrist tells him to “wish it away,” next night the psychiatrist sees the monster, who says, “Mervin sent me.  I hope it’s all right.” Now nobody else has to read it.  One star.

Race-Zoology and Politics, by Leon E. Stover

The “Science of Man” article by Leon E. Stover suffers the faults of its predecessors, magnified.  Race-Zoology and Politics is an outright polemic, with Stover taking up the cause of Carleton S. Coon, author of The Origin of the Races, who was denounced as a racist a few years ago by the president of the American Anthropological Association.  Stover says Coon “has simply become a ‘non-person’ to the profession,” but: “It is a dead certainty that Coon sometime in the future will be rehabilitated and recognized for the great work he has done, which has been to complete the uncompleted work of Darwin.”

Well, maybe.  Stover proceeds to argue Coon’s case about the evolution of human physical types in his familiar assertively dogmatic fashion.  This one-sided partisan presentation concerning what is apparently a hot ongoing argument in the profession is of little use to the lay reader trying to understand more about the underlying science.  Not rated—it’s just out of place here.

Summing Up

This is the most discouraging issue of Amazing in recent memory.  The magazine continues to limp along under the weight of the reprint policy, and this issue’s batch of them is the worst in some time.  Notably, the original notion of reacquainting the current SF readership with forgotten classics of the field—or at least interesting period pieces—has largely been lost as the reprints have come more frequently from Amazing’s more recent periods of outright mediocrity, mostly ranging from routine to awful.  Will yet another new editor be allowed to make it better?






[November 10, 1968] Ratings (December 1968 Fantastic)


by Victoria Silverwolf

Alphabet Soup

On the first day of this month, a new movie rating system created by the Motion Picture Association of America (MPAA) went into effect. Although the system is voluntary, filmgoers in the USA can expect to see a letter of the alphabet accompanying almost every movie.

This is very old news to those living in the United Kingdom, where a similar system has been in place since 1912. There have been some changes over the years, but currently the British ratings are:

U for Unrestricted (everybody admitted)

A for Adult content (children under 12 must be accompanied by adults)

X for Explicit content (no one under 16 admitted)

The new American system uses different letters, although they kept the scary X.

G for General audiences (everybody admitted, no advisory warnings)

M for Mature audiences (everybody admitted, but parental guidance is advised)

R for Restricted (persons under 16 not admitted without adult parent or guardian)

X for Explicit (no one under 16 admitted)

Gee, Magazines R Xciting!

In the spirit of the MPAA, let me experiment with offering my own similar ratings for the stories in the latest issue of Fantastic, in addition to the usual one-to-five star system of judging their quality.


Cover art by Johnny Bruck.

As with previous issues, the cover art for this one comes from the German magazine Perry Rhodan.


Hell Dance of the Giants, or something like that.

The fine print under the table of contents reveals that former editor Harry Harrison is now the associate editor, and former associate editor Barry N. Malzberg (maybe better known under the authorial pen name K. M. O'Donnell) is now the editor.  I have no idea if this swapping of job titles really means anything.

The Broken Stars, by Edmond Hamilton


Illustrations by Dan Adkins.

As the cover states, this is a sequel to Hamilton's famous space opera novel The Star Kings, from 1949. (I believe there have been a couple of other yarns in the series, published in Amazing.) However, it's certainly not a short novel. By my reckoning, it's a novelette, not even a novella.

I haven't read The Star Kings (mea culpa!) so it took me a while to figure out what was going on. (The fact that several paragraphs near the start are printed in the wrong order doesn't help.)

Three guys escape from a planet in a starship stolen from aliens. One fellow is the main hero, a man of our own time who somehow wound up in a far future of galactic empires and such. Another is a man of that time. So is the third one, but apparently he used to be the Bad Guy in previous adventures. Now he's working with the two Good Guys for his own self interest.

It turns out there's an alien on the ship as well. It can control human minds, but only one at a time. The trio solves this problem by crashing into a planet.


Out of the frying pan and into the fire.

The place is inhabited by nasty winged reptile aliens, who are part of an army of various extraterrestrials being collected by a Bad Guy to invade a planet ruled by the woman our time-traveling hero loves. Can he find a way to save her? Can he trust his former enemy? And what about those pesky mind-controlling aliens? Tune in next time!

This slam-bang action yarn reads like a chapter torn out at random from a novel. Besides starting in medias res, it stops before reaching a final resolution.

Hamilton is an old hand at writing this kind of space opera (they don't call him The World Wrecker for nothing!) so it's very readable. The former Bad Guy is the most interesting character (and he seems a lot smarter than the two Good Guys.) Too bad the story doesn't stand very well on its own.

Three stars.

Rated G for Good old scientifiction.

Ball of the Centuries, by Henry Slesar

Here's a brief tale about a guy who uses a crystal ball to see into the future. He warns a couple about to get married not to go through with it. Of course, they don't listen to him. Years later, they have the argument he predicted. The husband tracks down the guy and finds out the real reason he warned them.

That sounds like a serious story, but it's really an extended joke, with a double punchline. It's OK, I suppose, but nothing special, and a very minor work from a prolific and award-winning writer of fantasy, mystery, television, and movies.

Two stars.

Rated M for Matrimonial woes.

The Mental Assassins, by Gregg Conrad


Cover art by H J. Blumenfeld.

From the pages of the May 1950 issue of Fantastic Adventures, this story is the work of Rog Phillips under a pseudonym.


Illustration by Harold W. McCauley.

People who have been horribly maimed in accidents are kept alive and made to experience a shared dream world. The trouble begins when three of the twenty people develop evil alternate personalities. (As usual, the story thinks that schizophrenia literally means split personality.)

The physician in charge of the project asks the hero to enter the dream world and kill these doppelgängers. (This won't actually harm the real people, just eliminate their imaginary wicked doubles.) He gives it a try, but finds the experience so unpleasant he backs out of the deal.

The story then turns into a sort of hardboiled crime yarn, as the hero gets mixed up with a couple of mysterious women, a hulking bouncer, and two cab drivers who know more than they should. A wild back-and-forth chase ensues, partly on a spaceship, followed by a double twist ending.

You may be able to tell what's really happening as soon as the hero exits the dream world, but I don't think you'll guess the other plot twist, which is rather disturbing. This yarn reminds me of Philip K. Dick's games with reality, although it's not quite as adept.

Three stars.

Rated R for Really shocking ending.

The Disenchanted, by Wallace West and John Hillyard


Cover art by Vernon Kramer.

This fantasy farce comes from the January/February 1954 issue of the magazine.


Illustration by Sanford Kossin.

The ghost of Madame de Pompadour shows up at the apartment of a publisher. Present also is the author of a novel about the famed mistress of King Louis XV. The ghost objects to what the writer said about her in the book, and demands that it not be printed. When the publisher refuses, she has her ghostly buddies uninvent things, leading to chaos.

Strictly aiming for laughs, this featherweight tale ends suddenly. As a matter of fact, because the usual words THE END don't appear on the last page, I have a sneaking suspicion part of the story is missing. [Nope. It's that way in the original, too! (ed.)] Be that as it may, it provides a small amount of mildly bawdy amusement.

Two stars.

Rated R for Risqué content.

The Usurpers, by Geoff St. Reynard


Cover art by Raymon Naylor.

The January 1950 issue of Fantastic Adventures is the source of this chiller by Robert W. Krepps, an American author hiding behind a very British pen name.


Illustration by Leo Summers.

The narrator is a one-armed veteran of the Second World War. An old comrade-in-arms shows up and tells him a bizarre story.

It seems the fellow recovered from a serious eye injury. When his vision was restored, he saw that about half the people around him were actually weird, horrifying monsters in human disguise. He reaches the conclusion that beings from another dimension are infiltrating our own, intent on displacing humanity.

Things go from bad to worse when some of the creatures realize the guy can perceive them. They try to kill him, while he destroys as many of them as he can, leading to the violent conclusion.

This shocker is most notable for the truly strange and creepy descriptions of the monsters, each one of which has a different form. As an ignorant American, I found it convincingly British, although somebody from the UK might disagree. Overall, a pretty effective horror story.

Three stars.

Rated R for Revolting creatures.

The Prophecy, by Bill Pronzini

Like Henry Slesar's piece, this is a miniscule bagatelle about a prediction. A prophet who is always right announces that the world will end at a certain time on a certain day. When the hour of doom arrives, the unexpected happens.

Even shorter than the other joke story, this tiny work depends entirely on its punch line. I can't say I was terribly impressed. I also wonder why the magazine printed two similar tales in the same issue.

Two stars.

Rated G for Goofy ending.

The Collectors, by Gordon Dewey


Cover art by Barye Phillips.

My research indicates that somebody named Peter Grainger is an uncredited co-author of this story from the June/July 1953 issue of Amazing Stories.


Illustration by Harry Rosenbaum.

A very methodical fellow, who keeps track of every penny, tries to figure out why a small amount of money disappears every day. He runs into a woman who experiences the same phenomenon. It seems to have something to do with a vending machine.

The editorial introduction dismissingly says this story is . . . no classic, to be sure, it isn't even a minor classic . . . which seems like an odd way to talk about something worth printing. I thought it was reasonably intriguing. In this case, the open ending seems appropriate.

Three stars.

Rated M for Mysterious conclusion.

Unrated

As I mentioned above, the MPAA rating system is voluntary.  No doubt a few movies will be released without one of the four letters.  In a similar way, the stuff in the magazine other than fiction isn't really appropriate for rating.

Editorial: The Magazines, The Way It Is, by A. L. Caramine

Brief discussion of the rise and fall of science fiction magazines, with an optimistic prediction that they're on the way up again.  A note at the end states that A. L. Caramine is the pseudonym of a well-known science fiction author.

Digging through old magazines, the only reference I can find to A. L. Caramine is as the author of the story Weapon Master in the May 1959 issue of Science Fiction Stories.


Cover art by Ed Emshwiller.

A glance at the magazine tells me that, in addition to a story by Robert Silverberg under his own name, there are book reviews by the same fellow under his pseudonym Calvin M. Knox.  Given the way that single authors often filled up magazines with multiple pen names, I suspect that the mysterious A. L. Caramine is Silverberg as well, although I don't have definite proof of this.

2001: A Space Odyssey, by Laurence Janifer

One page article that praises the film named in the title, and says that Planet of the Apes is lousy. Just one person's opinion, take it or leave it.

The Rhyme of the SF Ancient Author or Conventions and Recollections, by J. R. Pierce

Parody of the famous Coleridge poem mocked in the title. It says that science fiction writers shouldn't go chasing money by writing other kinds of stuff. Pretty much an in-joke, I guess.

Fantasy Books, by Fritz Leiber

Mostly notable for a glowing review of Picnic on Paradise by Joanna Russ. May be the best-written thing in the magazine!

Good? Mediocre? Rotten? Xcruciating?

All in all, this was a so-so issue. The two star stories weren't that bad, the three star stories weren't that good. Not a waste of time, but you might want to listen to the current smash hit Hey Jude by the Beatles instead.


David Frost introduces the Fab Four as they perform the song on his television program.

Rated G for Groovy.






[July 14, 1968] Long Time No See (August 1968 Fantastic)


by Victoria Silverwolf

Welcome Back, Comrade

It's been more than a quarter of a century since the Communist Party of the United States ran candidates for President and Vice-President. That was back in 1940, when Earl Browder and James W. Ford were nominated.


They didn't win.

This month, the Party chose Charlene Mitchell and Mike Zagarell for the honor.


Zagarell is technically too young to serve as Vice-President, but I don't think he'll have to face that problem.

Overdue Notice

One month isn't anywhere near as long as twenty-eight years, but the failure of a July issue of Fantastic to hit the shelves of drugstores and newsstands (in June, of course, given the proclivities of the publishing industry) may have caused as much anxiety among readers of imaginative fiction as the lack of a Commie candidate caused in Red voters.

Not to worry. My esteemed colleague John Boston has explained the situation in typically erudite fashion in his latest review of Amazing. I'll wait here while you go take a look.

Ready? Good. Now that we've got that out of the way, let's take a look at the August, not July, issue of Fantastic to see if our patience has been rewarded.


Cover art by Johnny Bruck.

Our first hint that the delay hasn't changed things very much, if at all, is the fact that the cover is once again a reprint from an issue of the popular German space opera serial Perry Rhodan.


The original always looks better.

The Two Best Thieves in Lankhmar, by Fritz Leiber

We begin in promising fashion with our old pals Fafhrd and the Gray Mouser, in another witty and imaginative adventure from the living master of sword-and-sorcery.


Illustration by Jeff Jones.

The two lovable rogues have gotten their hands on some incredibly valuable magic jewels. Each one of them tries to cash in on the stolen goods, making use of different fences.

The Mouser goes to a blind fellow, who has a nubile female assistant. Fafhrd seeks out a woman of mature years, who insists on an intimate encounter before the deal is completed. Suffice to say that things don't work out as they expect.

As you'd expect, this is a beautifully written and highly enjoyable tale. It's a bit lighter in tone that some other stories in the series; an anecdote rather than an epic, perhaps.

As a bonus, the likable character Alyx, created by Joanna Russ, makes a guest appearance. Obviously Leiber approves of the way Russ is influenced by his work, and he has acknowledged this in a gracious manner.

Four stars.

Fault, by James Tiptree, Jr.

A new writer makes his third appearance in print with this science fiction story. Narrated by a spaceman to an unknown listener over drinks, it tells how an inexperienced crew member got in trouble. It seems he clumsily injured an alien. Put on trial, he is found guilty and punished in a way the aliens can't convey to the humans. He seems perfectly fine, until strange things start happening.

What the aliens did to the fellow is the whole point of the narrative. It's pretty much a puzzle story. For that kind of thing, it's reasonably interesting. It could have appeared in Analog, except for the fact that the aliens aren't shown to be inferior to humans. It's not bad, but not outstanding in any way.

My advice to Mister Tiptree is to keep writing; the man shows promise.

Three stars.

Horror Out of Carthage, by Edmond Hamilton

Here come the reprints. This old-fashioned yarn comes from the September 1939 issue of Fantastic Adventures.


Cover art by Harold W. McCauley.

Our cast of characters includes the manly hero, an older archeologist, and the latter's beautiful daughter. They're on a dig to locate the Temple of Moloch at the site of Carthage.


Illustrations by Jay Jackson.

Right away we're told that the daughter feels as if someone is trying to force her out of her body. It's no surprise, then, when the mind of a woman of the ancient vanished city takes possession of her physical form. Pretty soon our hero's mind goes far back in time to inhabit the body of a Carthaginian man.

The big problem is that Carthage is about to be wiped off the map by invading Romans. (The two folks from the doomed city came forward in time to escape that fate.) Can the hero find a way to save his beloved from being sacrificed to Moloch, and return to his own time with her? Come on, you know the answer to that already.


War, with elephants.

This is a typical old-time pulp adventure story, with characters who are walking archetypes. It's got some vivid scenes, so it's not boring. Carthage is constantly described as a wicked, barbaric place. That sounds more like Roman propaganda than accurate ancient history, but I'm no expert.

Worth a look for nostalgia buffs.

Three stars.

The Supernal Note, by Rog Phillips

The July 1948 issue of Amazing Stories supplies this unusual work.


Cover art by Arnold Kohn.

A mysterious entity sends a musical note from an ethereal realm to the material world. In mundane reality, a man strikes up a conversation with an airline stewardess. They are obviously attracted to each other, but eventually go their separate ways.


Illustration by William A. Gray.

This is a very strange story, and I have described it badly. The author creates a highly detailed mythological background, much of it difficult to comprehend. I'm not really sure what he's getting at. Did the musical note cause the pair to fall in love?

I found this peculiar tale rather haunting, if confusing. It's definitely not the same old thing, anyway.

Three stars.

When Better Budgies Are Built, by Bryce Walton

The November 1952 issue of Fantastic Adventures is the source of this futuristic farce.


Cover art by Robert Gibson Jones.

The narrator is a vacuum cleaner salesman. He gets pulled into the future by a guy using a forbidden time machine. It seems that two rival merchandisers, the only ones left in this new version of the USA, are about to start selling gizmos that will supply everything that anyone could want, for a price. The problem is that one of the corporations has an army of robots who are able to sell anything to anybody.


Illustration by William Slade.

What makes this even more alarming is the fact that the head of the company is a would-be dictator planning to use the robots to sell people on the idea that he should be their leader. In exchange for a piece of future technology that will make him rich when he goes back to his own time, the narrator figures out a way to defeat the irresistible robot salesman.

Pretty silly stuff, really. The plot depends on the robots being absolutely perfect at selling merchandise and ideas, without any clue as to how they do this. We don't get to find out what the narrator earns for his service, either.

The ending makes use of a stereotype about women that is more goofy than offensive.

Two stars.

The Frightened Planet, by Sidney Austen

This two-fisted, he-man yarn comes from the October 1948 issue of Amazing Stories.


Cover art by James B. Settles.

A Cro-Magnon runs away from his tribe after a fight with the bullying leader. He witnesses a sphere arrive and discharge two men and a woman. After saving the trio from a wolf, he jumps into the vessel to escape a sabretooth tiger. The four go off to another planet.


Illustration by J. Allen St. John.

The folks on this world are under attack by green monsters. The Cro-Magnon defeats the creatures easily, while the effete males around him cower in fear. Naturally, the woman is instantly attracted to his manliness.

The author is obviously trying to promote the idea that men should be fearless warriors. The Cro-Magnon's contempt for the decadent males surrounding him is evident, and the author appears to share it.

Even if I ignore all that, as an adventure story it failed to hold my interest. There are parts of it where there seems to be something missing; one scene jumps to another without any kind of transition.

One star.

You Could Be Wrong, by Robert Bloch

Here's a tale of paranoia from the March 1955 issue of Amazing Stories.


Cover art by Ed Valigursky.

A guy gets fed up with everything being fake. He goes on and on about this, until his exasperated wife calls in a buddy to talk some sense into him.


Illustration by Virgil Finlay.

The two fellows argue about stuff being phony for a while. The guy reveals what he thinks is behind all these ersatz things. There's a twist ending you'll see coming a mile away.

Definitely a one idea story. It's like one of Philip K. Dick's what-is-reality tales, with all the subtlety and complexity surgically removed. Or maybe it's more like a clumsy version of Robert A. Heinlein's famous solipsistic nightmare They.

Anyway, not very good.

Two stars.

No Head for My Bier, by Lester del Rey

This screwball comedy comes from the September 1950 issue of Fantastic Adventures.


Cover art by Robert Gibson Jones.

A nutty scientist uses a gizmo to remove an actor's head, as far as anybody can tell. Apparently he can still talk and breathe and such. He tells the actor to get a job without his handsome face within a month, or stay that way forever.


Illustration by Robert Gibson Jones also.

The actor's head is stored, in some way or other, like a photographic negative. Only pure alcohol can make it go back to normal. Let's just say that beer and a cat are involved in the ridiculous climax.

This thing is even more of a lunatic romp than I have indicated. The nutty scientist does all kinds of impossible things, from teleportation to literally flying.

Of possible interest to fans of pure wackiness.

Two stars.

The Wrong People, by Ralph Robin.

Yet another comedy, from the November/December 1953 issue of the magazine.


Cover art by Vernon Kramer.

A married couple who pretty much dislike everything, including each other, inadvertently conjure up a being from somewhere else in space and time. The creature is friendly enough, it seems, even if it scares the daylights out of the humans at first.


Illustration by Ed Emshwiller.

After they calm down, they think it's some kind of genie or something, ready to offer them whatever they want. This misunderstanding doesn't end well, leading to a shockingly gruesome conclusion.

There seems to be a touch of satire here, although you have to dig deep to find it. The sudden change in mood at the end really threw me for a loop.

Two stars.

Edgar Rice Burroughs' The Princess of Mars, by Charles R. Tanner

The author retells the story of ERB's famous novel in the form of a humorous poem.


Illustration by Jim.

I found it too sophomoric for my taste in literary spoofing. I may be prejudiced, as I am not a fan of Burroughs.

One star.

Worth Waiting For?

This issue started off well, but quickly sank into mediocrity and lousiness. Amazing and Fantastic seem to have reached the bottom of the barrel when it comes to reprints. Too much thud-and-blunder adventure, too much stupid comedy. It's enough to make you sick.


Cartoon by Frosty, from the same issue as Ralph Robin's story.






[June 6, 1968] The Stalemate Continues (July 1968 Amazing)


by John Boston

This July Amazing—wait, what?  You thought Amazing appeared in even-numbered months?  No more.  The mis-dating of the April issue as June means that what was to be the June issue has been pushed back—or at least the cover date has been—to avoid the confusion and likely loss of display time and sales had the publisher released a second issue dated June.  And Fantastic is pushed from July to August to keep these bimonthly magazines in alternate months rather than in direct competition. 

This issue looks a little better than the last.  There’s a new and seemingly higher grade of paper; the pages look less pulpy and the magazine is a bit thinner.  The cover, by Johnny Bruck, is lighter and more attractive than his usual; even though there’s a line of guys waving ray guns, for the foreground he’s borrowed another sort of cliché from Ed Emshwiller—guy with firm jaw, determined expression, and clenched fist staring out towards the viewer, like he just stepped off an Ace Double.  Relatively speaking, it’s a relief.


by Johnny Bruck

Once more, all but one item of fiction are reprints, though this issue’s exception is more considerable than some: House A-Fire, by Samuel R. Delany, described as a short novel (at 33 pages!) on the cover and contents page, though editor Harrison acknowledges in the letter column that it is actually an excerpt from Delany’s new novel Nova, forthcoming from Doubleday.  Delany’s name is misspelled on the cover and contents page and in Harrison’s editorial, spelled correctly on the story’s title page and in the letter column.  Are you getting tired of all this nit-picking?  So am I.  But the persistent sloppiness of this magazine continues to irritate.

Editor Harrison, clearly chafing under the reprint regime, continues to tout the non-fiction contents (seemingly the only part of the magazine that he actually controls) on the cover—“New Feature by HARRY HARRISON” (an editorial) and “New Article by ROBERT SILVERBERG POUL ANDERSON and LEROY TANNER” (the book review column).” There are also a new “Science of Man” article by Leon Stover (see below) and a London and Oslo Letter by Brian Aldiss, recounting his travels in Scandinavia.  The book review column includes Robert Silverberg’s thoughtful review of Brunner’s new novel Quicksand, Poul Anderson’s slightly celebrity-struck review of Asimov’s Mysteries, and two reviews by “Leroy Tanner,” a Harrison pseudonym.  One is a perfectly reasonable review of James Blish and Norman L. Knight’s A Torrent of Faces.  The other, of Algis Budrys’s The Amsirs and the Iron Thorn, spends more space (about a page!) denouncing Budrys for his review in another magazine of a book Harrison co-edited than it does on Budrys’s book.  This is distasteful to read and represents notably bad judgment on the editor’s part.

Harrison’s editorial, titled The Future of the Future, picks up where last issue’s mistakenly truncated editorial left off, reiterating his division of the world into SF-1, SF-2, and SF-3, and proceeding mostly to a series of platitudes.  (“SF-3.  This is wide open now and there are no rules. No one school is SF-3 and no one particular style or clique is any more important than the others.”) He does amusingly recount that he asked J.G. Ballard to tell him what inner space is, and he was about to answer, but just then someone interrupted them and the answer never came.  The letter column, with its traditional title Or So You Say, is back as well, for those who care.

House A-Fire, by Samuel R. Delany

Delany’s excerpt House A-Fire is about a bunch of overprivileged kids who are seemingly able to gallivant around the galaxy at whim.  We first meet Lorq von Ray, son of a mining magnate in the Pleaides Federation (Earth is in Draco), as a child.  Lorq’s parents are big shots in local politics.  They vacation (or something) on an off-the-map world called Brazillia where things are a little primitive; one of the local amusements is a variation on cockfighting.  There, he meets two other children, Prince Red and his sister Ruby Red; their father, Aaron Red, is a hyper-wealthy spaceship mogul from Earth, proprietor of Red-shift Ltd. (I guess Acme was taken.) Prince has an artificial right arm and is belligerently sensitive about it. 


by Gray Morrow

Young Lorq is of course brilliant and among other things, when he’s a little older, has his own spaceship, which he races in the New Ark regatta, coming in second, before heading off to a party thrown by Prince on Earth—in Paris, at the Ile St. Louis.  (“Caliban can make Earth in three days.”) He and his crew arrive and Prince immediately recruits them to rescue Che-ong, “the psychodrama star,” and her hangers-on, who have gotten stuck in a snowstorm in the Himalayas and upon rescue, prove to be a bunch of stereotypically air-headed teenagers.

At the party, everyone must have masks, and Prince has prepared an elaborate pirate mask for Lorq.  Delany has hinted to the reader, but kept Lorq in the dark, about Lorq’s father being involved in piracy.  A bit later, Lorq encounters Ruby Red, who has gotten pretty grown up since last seen, and who lets him in on the joke.  Prince shows up and tells Lorq to get away from his sister, they have a fight, and Prince lays Lorq out and messes up his face with his prosthetic fist.  Lorq’s crew carries him away and Ruby shows up on the river in her skimmer-boat and takes them all to the spaceport.  Later, in a final scene, we see Lorq, now back home, rich, and scarred, and contemplating his future.

This all sounds in summary like an overripe pulp space opera, but it is framed in some striking visualization and writing, as one would expect from Delany.  Like Lorq’s first glimpse of the mature Ruby Red:

“Then there was this: her eyes were smashed disks of blue jade, her cheek bones angled high over the white hollows of her wide face.  Her chin was wide, her mouth thin, red, and wider.  Her nose fell straight from her forehead to flare at the nostrils (she breathed in the wind—and watching her, he became aware of the river’s odor, the Paris night, the city wind); these features were too austere and violent on the face of a young woman.  But the authority with which they set together would make him look again, he knew, once he looked away; make him remember, once he had gone away.  Her face compelled in the way that makes the merely beautiful sick with jealousy.”

Yeah, a bit hokey, but it’s good hokum, suitable to our modern age.  And keep in mind that this is obviously all stage-setting for what one can hope are more substantial doings in the novel it is mined from.  Four stars, optimistically.

Locked Worlds, by Edmond Hamilton

Next up, straight from the September 1929 Amazing Stories Quarterly, is Edmond Hamilton’s Locked Worlds, all 50 pages of it.  It’s a sort of mad scientist story.  Dr. Adams, head of Physics at Northeastern University (a real place!), brilliant but widely disliked, discovers that the seemingly loose electrons sometimes found in atoms are really evidence that matter partakes of two worlds; our world’s electrons going around in one direction, the other world’s going in opposite directions.  Room for everybody! 

The rest of the profession isn’t having it and mocks Adams, who is determined to show them and get his own back.  Shortly he disappears, leaving his apparatus and a pile of bluish clay behind.  His assistant Rawlins comes to narrator Harker with an awful suspicion—and the newspaper clippings to prove it, sort of—that Adams has fled to the other world and that he’s planning his revenge there (the clippings refer to large and small piles of blue clay found at various places around the Earth).  So what to do for Rawlins and Harker but reconstruct Adams’s apparatus, follow him into whatever world he’s gone to, and thwart him?

And so they do, finding themselves on a mostly barren world with a blazing white sun overhead and blue clay under their feet.  And then—the giant spiders attack! 


by Frank R. Paul

Now Hamilton does not seem just to be trading on arachnophobia here.  Going forward, he refers to these giant spiders as spider-men, and shows them with a fairly advanced civilization.  But still, they signify that a cliched plot is about to take off, featuring captivity, aerial escape, pursuit, return in force with Earth’s new allies the bird-men (the birds and spiders engage in a dogfight), confrontation with the mad Dr. Adams, some literal cliff-hanging, and the ultimate triumph of good over evil.

Well, that was tedious.  It’s not for lack of enthusiasm on Hamilton’s part.  A sample, as our heroes escape the spiders with Nor-Kan, the bird-man, in the latter’s aircraft:

“He whirled to the craft’s controls, opened its speed lever to the last notch, and sent the air-boat racing on toward the south in a burst of added speed.  The great flying-platforms swiftly leapt after us, hurtling through the air at immense speed and slowly drawing ever closer toward us moving obliquely toward our own course.  Closer they came, and closer, air-boat and flying-platforms cleaving the air at a velocity unthinkable; now we saw from the foremost of the platforms behind us a shaft of brilliant orange light that burned toward us at the same moment.  Nor-Kan swerved the air-boat to avoid it.  He turned toward us, motioned swiftly toward the long tube-like projector mounted on a swivel at the stern of our own air-boat, and which I had already noticed.

“ ‘The static-gun!’ he cried.  ‘There are a few charges left in it—try to stop them with it!’ ”

Back in 1929 that would have been enough to get everyone’s blood up.  But in this decadent age, hot pursuit by ray-bearing airborne spiders just doesn’t seem to make it any more.  Or maybe it would take Delany to bring the spider-men to life.  Two stars.

The Genius, by Ivar Jorgensen


Uncredited

The other reprints in this issue are all from the 1950s, which is not necessarily good news.  Ivar Jorgensen is present with The Genius, from the September 1955 Amazing, except that Mr. Jorgensen is not really present because he doesn’t exist, being a house name used variously by Howard Browne, Harlan Ellison, Paul W. Fairman, Randall Garrett, Robert Silverberg, and Henry Slesar.  It is alleged in some circles that Randall Garrett is the mystery guest this time.  The story is a caveman epic, about old Zalu, who is trying to prove he’s still worth feeding so his grandson Cabo won’t bash his head in to get rid of him.  His plan doesn’t work, but Zalu does something rather significant en route to getting his head bashed in.  It’s short, readable, and mildly amusing.  Three stars.

The Impossible Weapon, by Milton Lesser


by Julian S. Krupa

None of the above can be said about Milton Lesser’s The Impossible Weapon (Amazing, January 1952), which is the kind of silly finger-exercise fluff that filled the back pages of the lower-level SF magazines in the 1950s.  Earth is losing a war to the League (League of what?  I forget), and our hero Stokes has figured out how to counter their super-weapon, but no one will listen to him, so in cahoots with a spaceman he meets in the wake of a barroom brawl, he commandeers a spaceship and takes off and proves he can do it.  Yeah, that oversimplifies a bit, but mercifully.  Stokes’s invention is silly, as is the supposed scientific rationale for it, as are all the other events from the beginning of the story to the end, so much so that I can’t bear to recount them.  Read the damn thing yourself if you must.  One star, too generously.

This Is My Son, by Paul W. Fairman


by Tom Beecham

Paul W. Fairman’s This Is My Son is from Fantastic for October 1955, during his two-year absence from the editorial masthead of that magazine.  It too is pretty dreadful.  Protagonist Temple, a young physicist with a fixation on getting a son, and his new wife are trying to reproduce, without success.  Temple has a great career opportunity and signs a contract taking him to South America for five years.  Jill is not pleased.  She wires him four months later that his son is due in five months.  But he can’t go back under his contract and if he breaks it he’ll be blacklisted.  After the five years he heads home to meet his son, and everybody’s happy, until he finds the manufacturer’s receipt for the android child, and reacts xenophobically.  Jill slaps him across the chops and then leaves after telling him, double-edgedly, that the child is as human as he is.  So he’s miserable for years, finally begins to see the error of his ways and sends the kid a gift.  Then the kid lands in the hospital after saving a couple of other kids from a fire.  Temple beats it to the hospital, the kid’s on the brink, so he offers an “old-fashioned blood transfusion” instead of the bottled plasma the nurse is about to give him.  Curtain, music swells, everything’s going to be fine.  It’s ridiculously contrived, sentimental, and manipulative, but at least demonstrates a little more craft than The Impossible Weapon.  Grading on the curve, barely two stars. 

Killer Apes—Not Guilty! , by Leon E. Stover

After the last two I am definitely in the mood for the contentious Dr. Stover, whose “Science of Man” article, Killer Apes—Not Guilty!, is suitably abrasive.  He takes on Robert Ardrey’s best-selling African Genesis from a few years ago, and he clearly has been waiting for his chance.  Ardrey attributed the bloody-minded and -handed character of homo sapiens to the apes from whom we descended.  Not so, says Stover; the apes were peaceful vegetarians (though not averse to the occasional grub or worm mixed in with their roughage), and the next step up (homo erectus) were carnivorous browsers, not carnivorous hunters.  We sapiens achieved our predatory status all on our own. 

Along the way Stover asserts with confidence a great deal about such subjects as the effect of domesticating fire on prehistoric social life, though without much explanation of how the dots were connected.  But he is also happy to patronize those of a different view, such as Ardrey’s favorite, the distinguished Professor Raymond Dart, late of the University of Witwatersrand: “Everybody is more than willing to let the old gentleman play with his pet theory that Australopithecus stood up to adult baboons and clouted them with humerus bones taken from antelopes.  Few take it seriously.” Good times!  Three stars.

Summing Up

Once more, business as usual at Amazing: signs of editorial vitality struggling to be seen beneath the clammy wet blanket of the publisher’s reprint policy, against the backdrop of negligent or indifferent production.  The stalemate continues.






[July 24, 1967] Not Feelin’ Groovy (Famous Science Fiction #1-3)


by Mx. Kris Vyas-Myall

I guess it had to happen. I have reached the age of 34 and am annoyed by modern pop music. I should say this is one specific type. Not the experimental psychedelic sounds of Jimi Hendrix or Pink Floyd, nor the soulful songs of Gladys Knight or The Four Tops. No, I am referring to sickly sweet “flower music” that has come over from California.

Feelin Groovy Harper's Bizarre Album

I first noticed it with Jan & Dean’s Yellow Balloon, a song which makes nursery rhymes sound like The Rolling Stones.

Windy by The Association Single

This was followed by more creeping into the charts such as The Association apparently performing a weather forecast and Harper’s Bizarre doing two awful covers of already poor songs. Then the worst has now been appearing everywhere. Scott McKenzie’s San Francisco, which sounds less like a pop song as an advertising jingle for flora hats.

San Francisco by Scott McKenzie

So many of them are appearing on pirate radio now, apparently superseding the beats and blues sounds I have enjoyed over the last few years. Having to hear so many cloying horticultural tunes from groups like The Young Rascals, The Turtles and The Johnny Mann Singers, is enough to make anyone want to hide in the past.

Thankfully there is a new magazine just for that: Robert Lowndes' (of '50s magazine editing fame) new effort, Famous Science Fiction.

Famous Science Fiction

Famous is about 90% reprint and 10% new material. We are told the purpose is to bring to light pre-1938 stories that were well regarded but have since been out of print, whilst also bringing back an intermediate market for SF between Amazing and the comics.


Famous #1


Famous #1 Magazine Cover

The cover is not an original Finlay, rather a colourisation of a piece from 1962’s Amazing.

Original Image from 1962 Amazing
Artwork by Virgil Finlay

The Girl in the Golden Atom by Ray Cummings

Ray Cummings story is his first and indeed was well known. However, it has been reprinted many times.

Printings of Girl in the Golden Atom

It first appeared in All-Story Weekly in 1919 and most recently in the collection The Giant Anthology of Science Fiction. Whilst this last reprint was thirteen years ago, it doesn’t feel as hard to find as Lowdnes seems to intend.

Anyway, it concerns a chemist (named merely The Chemist) who recites to his friends how he created a powerful microscope allowing him to see objects smaller than ever before. Looking inside a gold ring he finds a woman sitting inside a cave. He develops chemicals to shrink and grow himself so he can enter this microscopic world. From here it proceeds into an adventure tale as he must save her nation from destruction at the hands of an invading force.

Although it is important to acknowledge this story is almost fifty years old, this still feels old-fashioned for the time, more Victorian than Post-War in style. Also, even for the 1910s, the science is ridiculous. For example, the golden ring world resembles Earth because it comes from Earth, whilst Martian atoms would resemble Mars.

All of this would be tolerable if it weren’t so dull. Large sections are just spent with The Chemist explaining dull details and his friends ejaculating in surprise between puffs of cigars. Journey to the Centre of the Earth this is not!

Two stars…just.

The City of Singing Flame by Clark Ashton Smith

Smith fits Lowdnes’ brief better as, unlike fellow Weird Tales writers Howard and Lovecraft, his reprints have largely been restricted to a couple of Arkham House collections. That is, except for the Singing Flame stories!

Reprints of Smith's City of the Singing Flame

City of the Singing Flame first appeared in Gernsbeck’s Wonder Stories in July 1931. It was then combined with its sequel, from November of the same year, (see below) in Tales of Wonder in 1940 whereupon it became a regular reprint, up to Derleth’s The Other Side of The Moon, which you can still get in paperback today.

In the narrative, Hastane has received the journal of author Giles Angrath, who recently disappeared along with artist Felix Ebbonly. In the journal’s account, Angrath is walking near his cabin when he steps into a mysterious stone circle and is transported to another place. He begins to explore the strange new land, encountering the beings that dwell there and their Singing City.

Comparing this to Golden Atom, Singing Flame does everything right Cummings' story does wrong. Where Atom gets bogged down in technical gobbledygook, this is just willing to say it doesn’t understand, whereas the former creates an unimaginative repetition of our world, the latter is a work of colossal imagination, unlike any other I have read. And, most importantly, it is never dull.

Four stars

Voice of Atlantis by Laurence Manning

Wonder Stories Cover 1934

This comes from Wonder Stories in 1934, but I do not believe it has been reprinted. Congratulations, one out of three!

Clearly a fan of Manning, Lowndes has reprinted three others from this series in Magazine of Horror, where members of The Strangers Club tell each other unusual tales.

Volking tells of his experiments in telepathy, where he makes contact with a man from twenty thousand years in Earth’s past. This is a man from Atlantis, whose civilization was significantly more advanced than ours and is surprised by how savage we are today.

Erewhon by Samuel Butler Cover

This also feels very Victorian, reminding me of lost civilization tales like Erewhon. It should also be mentioned that even the characters note its style of using Socratic dialogue feels clumsy and the science is nonsensical. At least there is a kernel of some interesting ideas.

Two Stars

And now for the two new vignettes:

The Plague by George Henry Smith

In 2200, The Death Thing has come to a convent to claim the lives of eighteen young women. If Father Joseph does not accede to the request, all the children may be taken.

This is a dark and grim, if rather obvious tale. Like a combination of Killdozer and a Twilight Zone episode. However, the atmosphere raises it up a little.

Three Stars

The Question by J. Hunter Holly

Fifty years ago, the Vegans first encountered humanity. Humanity was told they would be allowed to join the family of intergalactic civilization if Earth could wipe out warfare. Since then, the World has strived to reach that goal, but will the Vegans be satisfied?

Well told story, if a little old fashioned and moralistic.

Three Stars

So, a mediocre start, with the one standout tale you can pick up elsewhere for a few shillings. But will it get better?


Famous #2


Famous #2 Cover

Another Finlay cover, this time from 1958’s Fantastic.

1958 Fantastic Reprint Cover
Artwork by Virgil Finlay

Inside Lowndes does better in his aims, with none of these stories appearing since first publication:

The Moon Menace by Edmond Hamilton

Weird Tales Cover from 1927

The first reprint comes from the September 1927 issue of Weird Tales, penned by the prolific, and still writing, Edmond Hamilton.

In The Moon Menace, Dr. Howard Gilbert, a famous but reclusive scientist, receives televisual signals from the Moon. Most other scientists doubt Gilbert’s findings but when the Earth is plunged into total darkness, he may be the world’s only hope.

It starts as a clear imitation of War of the Worlds and is a pretty standard invasion story. Whilst it may not be the most original work it has some interesting elements and readable enough to keep me engaged.

Three Stars

Dust by Wallace West

Although unpublished before, this was apparently rejected for Weird Tales publication some years ago.

Ralph Marvin of the Inquirer is writing up a story on how humanity may die out, but is it already here in the air we breathe?

A very didactic tale, one that could have been a science fact article. However, in spite of stylistic issues, it is a meaty subject that it is good to see addressed in fiction.

Three Stars

The White City by David H. Keller, M.D.

Amazing 1935 Reprint Cover

Taken from May 1935’s Amazing, Keller gives us yet another disaster yarn.

Farmer John Johnson decides to build a small holding in the slums of New York and live self-sufficiently. He becomes quite a sensation in the city as an eccentric, but when a terrible blizzard hits the Big Apple, he may be the one hope the world has.

This is an odd piece; for the majority it appears to just be the tall tale of an eccentric farmer. Then it takes a hard left turn into the kind of story you would see in the lowest of comic books.

Two Stars, mostly for curiosity value.

Rimghost by A. Bertram Chandler

The other new tale is a further outing for Chandler’s Rim stories, which we have been covering from the early days of the Journey to the most recent serial in If.

Mr. Willoughby joins a motley crew aboard the Rimgirl. However, something strange occurs, they encounter an exact duplicate of their ship, including its crew.

This spends far too much time for me running through all the characters and establishing connections to other stories such that the actual mystery is treated too abruptly. And, whilst the actual prose is solid, the misogynistic descriptions of Mary are poor.

A low Two Stars

Seeds from Space by Laurence Manning

Cover Wonder Stories 1935

And we finish with another visit to the Strangers Club, this one getting the cover of Wonder Stories for June 1935.

This time Col. Marsh tells of Blenkins who grows plants on his roof in Greenwich village. He plants some strange seeds in this garden and they grow into unusual tall plants. Eventually they walk into his apartment, telling him they are an intelligent species.

A reasonable story of sentient plant life, but it is less Day of the Triffids and more a forgettable tall tale.

Two Stars

So, whilst no complete blunders this issue, no stand outs either. Will third time be the charm?


Famous #3


Famous #3 Cover

Our final issue goes further back for its cover, from Science Stories in 1953.

Reprint Image from 1953's Science Stories
Artwork by Virgil Finlay

Beyond the Singing Flame by Clark Ashton Smith

This picks up after the publication of Angrath’s journal, where Hastane goes in search of the mysterious city. Within it he encounters even more wonders and what becomes of people who go through the flame.

I feel much the same about this sequel as I do about its antecedent. It is not so much a new idea and largely concerned with continuing exploration of this world, but Smith is such a marvelous wordsmith, the sense of awe pulls you along.

Four stars

A Single Rose by Jon DeCles

The only new fiction this issue. Silas Finnegan is a successful industrialist, who uses all his resources to make the one thing he always dreamed of, his very own unicorn. Of course, he then has to work out how to afford to keep it.

This piece seems to be aiming for something deeper about the nature of beauty, but I mostly just found it a pleasant little story about achieving childhood fantasies.

Three stars

Disowned by Victor Endersby

Astounding 1932 Coverr

This one comes from September 1932’s Astounding, although reads to me more like something I would expect in Weird Tales.

On a rainy night a party is caught in a storm and Tristan is struck by ball lightning. This causes Tristan’s gravity to be reversed and he is being pulled upwards towards the sky.

Disowned Artwork
Artwork by H. W. Wesso

This is very silly, not just in the science, but also in the circumstances which follow from it, with him living an upside-down life on the ceiling and doing circus acts.

One Star

The Last American by J. A. Mitchell

This is the earliest story so far, originating in book form in the 19th Century. However, it is once again one that Derleth currently has out in paperback.

Far Boundaries Cover

By 1990, the Mehrikan civilization vanished from the Earth and remains a mystery to the historians in Persia. This recounts the voyage of Noz-yt-ahl aboard the Zlothub in 2951 to their mysterious land.

Last American Art, New York In Ruins
All Artwork also by the Author

Landing in a strange port with huge structures, they eventually ascertain it is the fabled lost city of Nuh-Yuk, where the people were famous for nothing but their greed and having only prosperity as their God. As they continue to explore Nuh-Yuk they become less enamoured with the civilization, finding the people and buildings monotonous.

Artist's Impression of Life in Nuh-Yuk

As such they then head down river to Washington and there encounter Jon and his family, the very last remnants of the Mehrikans.

Fight between the Persians and Mehrikans

In spite of its age, this holds up as a great satirical piece, with the American being put in the position of the fallen civilization, judged harshly by those in a now dominant position and treated as a museum piece.

A high four stars

The Man Who Awoke by Laurence Manning

Continuing his reprints of Manning’s back catalogue, Lowndes has moved on to his Man Who Awoke series, with this first part coming from March 1933’s Wonder Stories.

Wonder Stories March 1933

Norman Winters has discovered that by putting himself into a comatose state in a chamber protected from cosmic rays, he can survive without aging. Faking his disappearance, he then sets up an x-ray to wake himself up in the year 5000.

Winters wakes in a time of plenty but not much excitement. People live in small villages and get everything from the trees they grow, only working less than two hours a day. Once the truth about himself is revealed he is caught in struggle between the Oldsters and the Council of Youth. Eventually Winters decides he cannot live here and uses the same method to advance to a later time. To be continued.

News From Nowhere by William Morris Cover

When I started, I thought it was going to be another The Sleeper Awakes. However, it is actually closer to William Morris’ News From Nowhere, showing us an agrarian future without want or struggle, and also asks questions of our current waste of our natural resources.

But this is not a purely a utopian vision, it acknowledges that the level of reaction to “The Age of Waste” (as they call the 20th Century) has resulted in excessive caution and explicitly calls for a middle path, for progress to exist without careless consumption.

Also, in stark contrast to his Stranger Club tales, this is elegantly written. Rather than wading through treacle I felt like I was drinking a glass of dry white wine on a summer’s evening.

Five Stars

Finally, we get the reader rankings for the first issue here:

Reader Ratings Issue 1: 1) Golden Atom; 2) City of the Singing Flame 3) Voice of Atlantis 4) Question 5) Plague
Showing myself to have very different opinions from the average reader of this publication

Some Other Someday

West Coast Consortium Band Photo
West Coast Consortium, actually from London

So maybe the past isn’t always that amazing either. Whether you are looking at then or now, there will always be both muck and brass.

I am not sure if I will pick up future issues or stick to picking up paperback anthologies for my past exploration. But, even though I will not put plants on my head for a trip to America, I am still happy to listen to Radio London, maybe just turning down the volume if West Coast Consortium come on…






[March 6, 1966] Is More Less? (April 1966 Amazing)


by John Boston

Two Weeks in Philadelphia

“GIANT 40TH ANNIVERSARY ISSUE”
“BIG 196 PAGES”

These are the blurbs on the cover of the April Amazing.  Yeah, and W.C. Fields said, “Second prize is two weeks in Philadelphia.” After February’s dreary procession of the better forgotten from Amazing’s back files, the promise of an all-reprint issue with 32 more pages is dubious at best.  The architect Ludwig Mies van der Rohe likes to say, “Less is more.” We are about to test the converse hypothesis.


by Frank R. Paul, Robert Fuqua, and Hans Wessolowski

But first, the setting for this diamond.  You see the drab cover, with the collage of tiny reproductions of early Amazing covers crowded to the edge by a bulldozer of type.  Inside, besides the fiction, there is Hugo Gernsback’s editorial from the first issue of Amazing, no more interesting than you would expect, and a two-page letter column, which unlike prior columns includes a letter critical of the reprint policy.  More interesting and commendable is A Science-Fiction Portfolio: Frank R. Paul Illustrating H.G. Wells, seven pages of illustrations from early issues of Amazing featuring Wells reprints. 

But onward, to the fiction.  To begin, or to warn, I should note that much of this issue is dedicated to Big Thinks: the fate of humanity, the proper roles of the sexes in human society, and . . . class struggle!

Beast of the Island, by Alexander M. Phillips

Things begin reasonably well, and not too grandiosely, with Alexander M. Phillips’s Beast of the Island, from the September 1939 Amazing.  A couple of guys are plane-wrecked on an uninhabited Pacific island and discover there seems to be some large animal snuffling around—an animal that can talk, or try to.  On exploration, they find a cave, complete with ancient skeleton and trunk, which contains a journal detailing the failed struggle of some 17th century sailors to survive the attacks of this terrible beast, foreshadowing their own struggle.  This is a quite competent adventure story and the ultimate revelation of the nature of the beast (not to coin a phrase) is reasonably clever for its time.  Three stars.


by Robert Fuqua

The mostly-forgotten Phillips first appeared in Amazing in 1929 and published about a dozen stories in the SF/F magazines, the last in 1947.  Best known of these is probably his fantasy novel The Mislaid Charm, published first in Unknown, then in hardcover by Prime Press, one of the early SF specialty publishers.  He is also that unusual figure, a pro turned fan, having become a mainstay of the Philadelphia Science Fiction Society, which did not exist when he started writing. 

Intelligence Undying, by Edmond Hamilton

Edmond Hamilton’s Intelligence Undying, from the April 1936 issue, is in equal measure splendid and ridiculous.  The brilliant but elderly Doctor John Hanley, frustrated because life is too short to complete all the work he has imagined, has a solution: he orders up a newborn infant (prudently, a “white male child”) from the legions of abandoned children, and decants the contents of his brain into the child’s.  (Never mind that old country saying about trying to put ten pounds of . . . whatever . . . into a five-pound bag.) This kills the old Hanley, but he has named a young graduate student friend to be the child’s guardian.

That is an interesting set-up, but Hamilton immediately abandons it.  We flash forward to John Hanley the 21st, interrupted in his laboratory in the year 3144 because the rocket ships of the Northern and Southern Federations are fighting.  (“The fools, the blind fools!  After I’ve worked a thousand years and more to give them greater and greater powers, and they use them—.”) Soon enough the victorious Northerners show up to “protect” him, so he immobilizes them and the rest of the world by activating a device that disturbs their semi-circular canals so no one can stand up.  Hanley announces to the world that nations are abolished and he will be ruling them now.  Wounded, he orders the Northerners to go immediately and pick him up another male child.


by Leo Morey

Flash forward again to John Hanley 416, or the Great Jonanli, as he is worshiped worldwide.  The world’s population is idle, supported by the great automated factories Jonanli has established.  But now, he announces to the world, he has discovered that the Sun is about to collapse, rendering Earth uninhabitable.  There is nothing for it but to move to Mercury!  “There was stunned silence and then from the view-screens came back to him a tremendous, wailing outcry of terror. ‘Save us, Jonanli!  Save us from this death that comes upon us!’ ” He tells them that they’ve got to do some work to save themselves but just gets more wailing in return.

So the Great Jonanli reprograms (as our great scientists would put it today) all the auto-factories to crank out robots to build the spaceships, give Mercury some rotation (it was not known in 1936 that it does rotate), terraform it (as we put it today), build cities, and start plants growing.  “The humans of Earth helped in none of this but lay supine in terror, crying out constantly to Jonanli and staring in terror at the sun.”

As the sun visibly falters, Earth’s population is ushered onto the spaceships, ferried to Mercury, and dumped there by the robots, who then destroy themselves.  John Hanley stays on Earth awaiting the end and dies buried in snow, having learned his lesson, leaving humanity to figure out once more how to take care of itself.

Technological progress leading to stagnation and rebirth (or the lack of it) is of course one of the great themes of SF, both its regular practitioners and drop-ins like that E.M. Forster guy.  Here Hamilton renders it with studied crudeness, a comic book without the pictures, terror and majesty pitched to the guy reading the racing form on the subway, forget the Clapham omnibus.  Three stars for this absurd tour de force.

Woman’s Place

Two of the stories courageously address the question that haunts . . . somebody’s . . . mind: what is to be done about women—and before it’s too late!  Two tales of women-dominated societies probe this urgent question.

The Last Man, by Wallace West

Brightness falls from the air in Wallace West’s The Last Man (from the February 1929 issue); all ridiculous, no splendor, Sexists in the saddle, bad taste in mouth.  In the far future, men have been abolished.  “The enormous release of feminine energy in the twentieth to thirtieth centuries, due to the increased life span and the fact that the world had been populated to such an extent that women no longer were required to spend most of their time bearing children, had resulted in more and more usurpation by women of what had been considered purely masculine endeavors and the proper occupations of the male sex.


by Frank R. Paul

“Gradually, and without organized resistance from the ‘stronger’ sex, women, with their unused, super-abundant energy, had taken over the work of the world.  Gradually, complacent, lazy and decadent man had confined his activities to war and sports, thinking these the only worth-while things in life.

“Then, almost over night, it seemed, although in reality it had taken long ages, war became an impossibility, due to the unity of the nations of the earth, and sports were entered into and conquered by the ever-invading females.”

Artificial reproduction was developed and “the men were dispensed with altogether,” except for a few museum specimens.  Later: “In the ages which followed, great physiological changes took place.  Women, no longer having need of sex, dropped it, like a worn-out cloak, and became sexless, tall, angular, narrow-hipped, flat-breasted and un-beautiful.”

So here we are with M-1, the Last Man, physically a throwback (i.e., pretty hunky), who lives in a (rarely visited) museum with a caretaker, and is obliged to put himself on display in a glass cage one day a week for the benefit of women who want to gawk at this freak.  These women are “narrow-flanked flat-breasted workers, who stood outside the cage and gazed at him with dull curiosity on their soulless faces.”

But there’s an exception—an atavistic woman, conveniently telepathic, who shows up one night outside the glass cage, having slipped away from her keepers: “Hair red as slumberous fire—eyes blue as the heavens—a face fair as the dream face which sometimes tortured him.” Later: “her face assumed a faint pink tinge which puzzled him, yet set his pulses throbbing.” She calls herself Eve, and of course decides to call him Adam.  M-1 is horrified and fascinated, and slowly comes around to her rebellious point of view as she shows him around and takes him covertly to the birth factory, which has replaced cruder forms of reproduction.  Eve broaches the idea that they might escape and restart humanity the natural way. They are discovered, flee, and Eve hides in the museum and shares his rations.

In the museum, they find a large quantity of TNT, and hatch their plot to destroy the birth factory.  Afterwards, as they escape in a flying car, heading for the mountains, “the first rays of the rising sun splashed into the cockpit a shower of pale gold,” and never mind that they have just destroyed the prospects of a society of millions of people, like it or not.

So: women, if they don’t have to spend all their time minding children, will take over the world of work, and then somehow push men out of the world of sport (“sports were entered into and conquered by the ever-invading females”), and kill almost all of the men, and then (despite the earlier talk of “feminine energy”) create a stagnant, joyless, and regimented world in which progress has ceased and all but a few must spend twelve hours a day in tedious labor.  Whoa!  Guess we better keep them barefoot and pregnant!  Sounds like the author’s unconscious taking out its garbage.  One star, and a coupon good at any psychiatrist’s office. 

Pilgrimage, by Nelson Bond

Nelson Bond’s Pilgrimage offers a more genial take on the evils of matriarchy—that is, with less unalloyed misery on display than in The Last Man.  This story is said to be revised from its first appearance as The Priestess Who Rebelled in the October 1939 Amazing


by Stanley Kay

Civilization has fallen, and in the Jinnia Clan (not far from Delwur and Clina), the Clan Mother is in charge—of the warriors, with (like Wallace West’s future women) “tiny, thwarted breasts, flat and hard beneath leather harness-plates”; the mothers, the “full-lipped, flabby-breasted bearers of children . . . whose eyes were humid, washed barren of all expression by desires too often aroused, too often sated.” Then there are the workers: “Their bodies retained a vestige of womankind’s inherent grace and nobility. But if their waists were thin, their hands were blunt-fingered and thick.  Their shoulders sagged with the weariness of toil, coarsened by adze and hod.”

And there are the Men, with their “pale and pitifully hairless bodies,” not to mention their “soft, futile hands and weak mouths”; apparently they are in short supply and excluded from all useful activity except breeding.  There are also Wild Ones, rogue unattached males who want nothing more than to get their hands on Clan women and have their way with them.  They are sometimes recruited to join Clans, but their supply is dwindling too.

Our heroine, young Meg, has just hit puberty, and doesn’t much like the prospects she sees around her.  Nothing will do but to be a Clan Mother herself.  And with no hesitation, the wise and learned Clan Mother takes her on.  Meg learns “writing” and “numbers” and is introduced to “books.” But before she’s ready to roll as a Clan Mother, she’s got to go on her Pilgrimage to the Place of the Gods, far west and to the north.  She’s made it past the “crumbling village” of Slooie and into Braska when she is attacked by a Wild One, but saved by someone unexpected—Daiv of the Kirki tribe, “muscular, hard, firm,” who quickly tells her twice that she talks too much, and suggests that she mother a clan with him.

Daiv is quickly dismissed, and Meg sets out again, on foot, because her horse ran away during the affair of the Wild One.  But Daiv shows up again and introduces her to “cawfi,” and also to kissing.  “Suddenly her veins were aflow with liquid fire.”

At last, after the long journey northwest from Jinnia, she arrives at the Place of the Gods, and there they are: “stern Jarg and mighty Taamuz, with ringletted curls framing their stern, judicious faces; sad Ibrim, lean of cheek and hollow of eye; far-seeing Tedhi, whose eyes were concealed behind the giant telescopes.” The Gods are Men!  Real men, like Daiv!  What to do?  Return to the sterile and diminishing life of the Clan?  No!  She heads “back . . . back to the fecund world on feet that were suddenly stumbling and eager.  Back from the shadow of Mount Rushmore to a gateway where waited the Man who had taught her the touching-of-mouths.”

This of course makes very little sense, to send the Clan Mother-in training off on a pilgrimage that will undermine the entire basis of the society she is supposed to preside over, but that lapse of logic would seem to be beside the author’s urgent point.  Two stars; it’s less unpleasant than The Last Man

White Collars, by David H. Keller

White Collars, by David H. Keller, M.D., from the Summer 1929 Amazing Stories Quarterly, is a social satire, of sorts.  Keller was known for absurd extrapolation.  His most famous story may be Revolt of the Pedestrians, in which humanity has evolved, Morlocks-vs.-Eloi style, into automobilists (of cars and powered wheelchairs), whose legs have atrophied, and back-to-nature pedestrians, and of course they struggle for supremacy. 


by Hynd

Here, the trend towards more education for everybody has resulted in a huge oversupply of the college and professional school graduates, who are all too ready to remove your tonsils or teach you Greek, if only more people needed those services.  These White Collars, who are on the march with picket signs as the story opens, demand employment fitting their educations, and refuse to perform any of the practical work that is actually needed or accept the decline in social status that would go with it.  They’d rather live in desperate but genteel poverty and complain about it. 

The story consists largely of conversations between Hubler, a millionaire plumber, and Senator Whitesell, who is in the dam-building business but (as he puts it) “bought a seat in the Senate,” encouraged by his business associates, who “felt that our group was not being properly cared for.” (It’s hard to tell if this too is satire, or if everyone was a little less subtle about these things in Keller’s day.) Hubler takes Whitesell on a tour of the White Collars’ neighborhood, including a visit to the Reiswicks, the family whose daughter Hubler’s son is in love with.  The family will have none of an offer of productive but lower-status work and the daughter will have nothing to do with the son of a plumber. 

Senator Whitesell goes back to Washington, and the general problem is resolved with draconian legislation providing for involuntary servitude, complete with labor camps, and suppression of criticism.  This does wonders for formerly idle intellectuals: “They became different men and women, they sang at their work, and the number of babies born in the Labor Hospitals to happy mothers and proud fathers steadily increased.” The private problem of the Reiswicks is solved by a combination of emigration and the last-minute kidnapping and forced marriage of their daughter to the plumber’s son—but she decides she likes the idea after she sees his modern kitchen.

This of course is all mean-spirited and reactionary, as well as ridiculous, but hey, it’s satire, though Keller is no Jonathan Swift.  (And I wonder what Keller had to say a few years later about the New Deal.) Keller is at least a competent writer.  So, two stars, barely.

Operation R.S.V.P., by H. Beam Piper


by Robert Jones

Between West and Keller, we have a brief respite from gravity in H. Beam Piper’s Operation R.S.V.P., from the January 1951 issue, which presents the lighter side of the struggle for world domination.  Piper at this point had published several solid and well-received stories in Astounding, still one of the field’s leaders.  This one is flimsier: an epistolary story, told in memos among the Union of East European Soviet Republics and the United People’s Republics of East Asia, which are engaging in nuclear saber-rattling, and Afghanistan, which is outsmarting them both.  It is clever and well-turned and not much else; it aspires to little and achieves it handily.  Two stars.

The Voyage That Lasted 600 Years, by Don Wilcox

Don Wilcox, whose actual name is Cleo Eldon Wilcox, but who has also appeared as Buzz-Bolt Atomcracker (in Amazing, May 1947, for Confessions of a Mechanical Man), published SF from 1939 to 1957, almost entirely in Amazing and its companion Fantastic Adventures, mostly in the Ray Palmer era.  The Voyage That Lasted 600 Years, from October 1940, is a fairly well-known if not much-read story, chiefly because it was the first to explore the idea of a generation starship, preceding and possibly inspiring Robert Heinlein’s much more famous Universe.


by Julian Krupa

The good ship S.S. Flashaway carries 16 couples, plus the narrator, Prof. Grimstone.  He will serve as Keeper of the Traditions, traveling in suspended animation and being revived every hundred years to keep things on track, handily providing a viewpoint character for this centuries-long story.  Upon his first revival, he hears many babies crying; there is a population crisis.  Why?  Boredom, apparently.  Grimstone suggests wholesome activities: “Bridge is an enemy of the birthrate, too.” But alas: “The Councilmen threw up their hands.  They had bridged and checkered themselves to death.”

Solutions?  One character says, “We’ve got to have a compulsory program of birth control.” Prof. Grimstone in his recommendations “stressed the need for more birth control forums.” Not to be indelicate, but I don’t think people trying to avoid pregnancy use a forum.  And you’d think the people planning this trip would have made some provision for it—maybe even something futuristic, like, oh, a pill that would suppress ovulation or fertilization.  But I guess you couldn’t really talk much about that in a family magazine in 1940.

So, leap forward 100 years, and Grimstone awakes to find people lying around starving.  Babies are still the problem.  These people were born outside the quota, and by decree are not allowed to eat regularly.  Grimstone sets matters straight: everybody eats, there’s a new regime, everybody outside the quota is surgically sterilized, and inside the quota they’re sterilized after the second child.  And they’re all happy about it.

A century later, there’s no population problem, but factions are at each other’s throats, and Grimstone has to make peace.  And it goes on, century by century.  Wilcox has put his finger on the central problem of the generation ship idea: there’s no reason for the intermediate generations, who didn’t sign up for life in a big tin can and have nothing else to look forward to, to remain loyal to the mission and to keep the discipline necessary for a small community to survive for centuries.

There’s a pretty decent story here, unfortunately swathed in wisecracking Palmerish pulp style—the first line is “They gave us a gala send-off, the kind that keeps your heart bobbing up at your tonsils,” and that’s pretty representative.  It’s also weighed down by the taboos of the time in the overpopulation episode.  Wilcox gives the impression of a writer of limited gifts struggling to do justice to a substantial theme, which is both refreshing and frustrating.  Three stars, for effort and for originality in its time.

The Man from the Atom (Sequel), by G. Peyton Wertenbaker

The issue closes with G. Peyton Wertenbaker’s The Man from the Atom (Sequel)—yes, that’s the title—from the May 1926 Amazing.  You will recall that the narrator Kirby was invited over to Dr. Martyn’s place to try out his expander/contractor, pushed the Expand button like any good SF mark-protagonist of the 1920s and ‘30s, and found himself growing so large that his feet slipped off Earth and he wound up in a super-cosmos in which our universe was but an atom, trillions of years in the future.  He’s not thrilled about it, either. 


by Frank R. Paul

But he works the Shrink button and gets himself sized to land on another planet, thrusting his feet through the clouds as he downsizes.  There he falls into the hands of supercilious humanoids who imprison and interrogate him, but shortly the beautiful Vinda—daughter of the King of the planet, of course—shows up, providing “endless days of wonder and enchantment” (not biological, we are assured), and also offering a way back.  Well, not exactly back.  The way back is forward, because (after invocation of Einstein and the curvature of space), “the whole history of the universe is rigidly fore-ordained, and so, when time returns to its starting point, the course of history remains the same.” More or less, anyway.

So the humanoids make some calculations, he pushes the Expand button again, and before long arrives on (a slightly different) Earth, only to learn that Dr. Martyn has been imprisoned for murder after his disappearance, or rather, the disappearance of the corresponding Kirby in this world.  Now he's released, of course.  But after a while, home, or near-home, is not enough for Kirby; he pines for Vinda; and soon enough he is pushing the Expand button again, hoping to rejoin her in the next cycle of the universe, even if he has to fight the other version of himself that this cycle’s Dr. Martyn has previously dispatched.

This sequel is a noticeably higher class of ridiculous than its forerunner, better written and with considerably more ingenuity of detail along the way, so it laboriously climbs to two stars.

And I Only Am Escaped Alone To Tell Thee

Well, it could have been worse.  Two of these stories, Beast of the Island and, barely, The Voyage That Lasted 600 Years, are actually worth reading for reasons other than laughs or historical interest.  The rest are not, except for the overdone spectacle of Intelligence Undying.



[Don't miss TODAY'S episode of the Journey Show, starting at 1:00 PM Pacific — we have an all star cast of artists who will be doodling to YOUR specification.

Y'all come!]




[November 12, 1965] Doldrumming (December 1965 Amazing)


by John Boston

Off Days

The December Amazing, boasting Cordwainer Smith, Murray Leinster, Edmond Hamilton, Robert Sheckley, and Chad Oliver, looks promising despite the hideous front cover by Hector Castellon.  Unfortunately, the unifying theme of the issue is Off Days of Big Names.


by Hector Castellon

But first, let’s survey the terrain.  The Smith and Leinster stories are new, and informed rumor has it they are the first purchases of the new editor after the exhaustion of Cele Lalli’s leavings.  They are long, so the three reprints make up a smaller proportion of the magazine than in the previous issue, less than half of the total page count.  Almost all the the issue’s contents are fiction.  The editorial is one page, as is the letter column, and that’s it: no article, no book reviews.

The editorial by Joseph Ross cocks a fairly vapid snook at outside critics of SF, most recent example being Kurt Vonnegut, who isn’t entirely outside, and the letter column—both the letters and the editor’s responses—are calculated to cheer on the magazine and celebrate the true pulp quill, with a sideswipe at the previous editor’s attempts at something a little more elevated.

Killer Ship (Part 2 of 2), by Murray Leinster

The longest item is the conclusion of Murray Leinster’s serial Killer Ship, which inhabits the subgenre of Reactionary Science Fiction.  This is not a political designation, but a description of stories that suggest—nay, insist—that the future will, conveniently for the lazy reader and writer, not be much different from the past.  This one began last issue with: “He came of a long line of ship-captains, which probably explains the whole matter.”


by Norman Nodel

There follows a genealogy of the protagonist Captain Trent’s space- and sea-faring ancestors back to the eighteenth century, followed by several paragraphs about the similarity between the dangers of space travel and those of eighteenth-century sea voyaging, complete with Trent’s ancestor sailing into port with the hanged bodies of pirates swinging from the yardarms.  There’s no indication of what Trent knows or how he has been influenced by these ancestors’ doings, so how his lineage “explains the whole matter” is a bit murky.

A couple of pages later, after it is disclosed that the ship-owners who have hired Captain Trent for a trading voyage in pirate-infested waters, er, space, would be just as happy if he gets pirated so they can collect the insurance: “It didn’t bother him.  He came of a long line of ship-captains, and others had accepted similar commands in their time.”

Six pages further on, when it appears Trent’s ship has spotted a lurking pirate: “The report of a reading on the drive-detector was equivalent to a bellowed ‘Sail ho!’ from a sailing brig’s crosstrees.  Trent’s painstaking use of signal-analysis instruments was equal to his ancestor’s going aloft to use his telescope on a minute speck at the horizon.  What might follow could continue to duplicate in utterly changed conditions what had happened in simpler times, in sailing-ship days.”

Later still: “The arrival of the Yarrow in port on Sira was not too much unlike the arrival of a much earlier Captain Trent at a seaport on Earth in the eighteenth century.” I will spare you the extensive elaboration.  And I can’t resist one more, towards the end as the Captain and his men are mustering for the final battle: “When they gathered, crowding, to get into the Yarrow’s spaceboats, the feel of things was curiously like a forgotten incident in the life of a Captain Trent of the late eighteenth century.” (Again, spare the details.) There is no suggestion that the current Captain Trent is in any way aware of this incident.  Hey, the author just said it’s forgotten!

At this point it is tempting to ask, Why bother?  Why not just swing by the library and pick up a stack of old C.S. Forester novels, and take your eighteenth century straight?

Another conspicuous feature is its pervasive verbosity.  Consider the following passage, right after the discovery that there’s another spaceship lying low very close.  Trent throws a switch that turns on the signal-analyzing instruments and goes to work.  Now:

“There was silence save for that small assortment of noises any ship makes while it is driving.  It means that the ship is going somewhere, hence that it will eventually arrive somewhere.  A ship in port with all operating devices cut off seems gruesomely dead.  Few spacemen will stay aboard-ship in a spaceport.  It is too still.  The silence is too oppressive.  They go aground and will do anything at all rather than loaf on a really silent ship.  But there were all sorts of tiny noises assuring that the Yarrow was alive.  The air apparatus hummed faintly.  The temperature-control made small, unrelated sounds.  Somewhere somebody off-watch had a tiny microtape player on, the Aldonian music too soft to be heard unless one listened especially for it.”

Next: “The signal-analyzer clicked.” Intermission over!  Story starts up again! 

And here’s another one, short but telling.  Captain Trent and the captain of a pirate-bashed ship whose crew Trent has rescued are about to travel from one ship to the other.  “The Yarrow’s bulk loomed up not forty feet away, but beneath and between the ships lay an unthinkable abyss.  Stars shown up from between their feet.  One could fall for millions of years and never cease to plummet through nothingness.” Then they snap on lines and are hauled across the 40 feet, sans plummeting or any actual risk or fear of it.

A little later (we’re up to page 29 of the October issue), there is a long description of the pirates repairing the damage to their ship that Captain Trent inflicted by ramming them.  This is actually a nice vivid word-picture.  But then:

“While this highly necessary work went on, the stars watched abstractedly.  They were not interested.  They were suns, with families of planets of their own; besides, some of them had comets and meteoric streams and asteroid belts to take up their attention.  There was nothing really novel in mere mechanical repair-work some thousands of millions of miles away from even the nearest of them.”

And it goes on, and on, appearing everywhere like water seeping up through the floorboards of a flooding house.  It’s enough to make a body wonder if paying by the word is really such a good policy.

Oh, yes, there is also a story here, fitfully visible through the padding and the constant eruptions of the eighteenth century.  Trent takes on a job carrying a cargo through pirate territory, partly to make some money and party because he hates pirates.  He has an encounter with some pirates, captures some of their crew, and rescues the boss’s daughter (boss meaning owner of the pirated spaceship, and also a planetary president).  She thinks he’s the cat’s meow for rescuing her, and he sort of likes her too, but duty calls.  Then everybody foolishly thinks it’s safe to travel again because Trent defeated this lot of pirates.  The boss’s daughter gets kidnapped by pirates again.  Trent cleverly figures out where she and the other hostages must be, goes there with his crew, confronts the pirates in their lair, rescues boss’s daughter again, wedding bells clearly to follow. 

There are some clever plot twists along the hackneyed way, as one would expect from a guy who’s been at this for well over four decades.  There are also characters, sort of.  Captain Trent is the strong laconic guy who may have inner turmoil but keeps it to himself.  Everybody else is essentially a cartoon, notably Trent’s crew, who play a big part in his success, and who are essentially a bunch of roughnecks the Captain has recruited from barroom brawls and who follow him because he’s a pretty good brawler too.  Finally, there is the definitive happy ending: “This novel will be published in the winter by Ace Books under the title ‘SPACE CAPTAIN.’

One star for both parts.  That’s the average of two stars for smooth professionalism, and zero stars for polished vacuity; life’s too short to waste time on this.

On the Sand Planet, by Cordwainer Smith

All right, Henry, wheel that one out and release it to the next of kin.  Who’s on the next slab?  Oh, Cordwainer Smith.  Sounds promising.  Except . . . 

On the Sand Planet seems to be the last in the Instrumentality series featuring one Casher O’Neill that began with On the Gem Planet and On the Storm Planet, with Three to a Given Star tangentially related.  They were all published in Galaxy, to considerable praise from the Traveler.  But . . . if the others appeared in Galaxy, what is this one doing here at the bottom of the market?  Unfortunately, suspicions confirmed.


by Jack Gaughan

Casher O’Neill has been on a mission to relieve his home planet Mizzer of the tyrant Wedder, and to that end has circuitously toured the galaxy and has obtained various superpowers, apparently courtesy of T’ruth, an Underperson derived from a turtle.  That’s all before this story opens.  Now, he’s landing on Mizzer again, walks into town and into Wedder’s citadel, and using his superpowers, rearranges Wedder’s head and portions of his supporting anatomy, turning him into a pussycat.  Metaphorically, I mean.  While he’s at it, Casher restores the intelligence of an idiot child. 

Now that Casher is done with his life’s work, he drops in on his mother, who has mixed feelings about him, and his daughter, who has her own life and would just as soon he went away.  So he decides to go to the Ninth Nile (this city Kazeer is at the confluence of a whole lot of Niles, it seems), though he is warned he will need iron shoes for the volcanic glass.

At the Ninth Nile, Casher meets D’alma, an elderly dog-underperson and an old acquaintance, who accompanies him, first to the gaudy City of Hopeless Hope, where everyone seems to be engaged in the practice of one religion or another, and D’alma warns that they are “the ones who are so sure that they are right that they never will be right.” Then, to the place of the Jwinds, “the perfect ones,” who destroy intruders who don’t meet their high standards.  But Casher, who contains multitudes in his enhanced cranium, is too much for them.  On to Mortoval, where a gatekeeper lets them pass when Casher again musters his superpowers to invoke “old multitudes of crying throngs.” The gatekeeper asks, “How can I cope with you?”

“ ‘Make us us,’ said Casher firmly.
“ ‘Make you you,’ replied the machine.  ‘Make you you.  How can I make you you when I do not know who you are, when you flit like ghosts and you confuse my computers?’ ”

On to Kermesse Dorgueil, where D’alma warns “here we may lose our way because this is the place where all the happy things of this world come together, but where the man and the two pieces of wood never filter through,” and a guy named Howard explains, “We live well here, and we have a nice life, not like those two places across the river that stay away from life,” and they make no claim to perfection. 

Here Casher encounters a woman, Celalta, who is dancing and singing, having resigned as a lady of the Instrumentality, and Casher recruits her as traveling companion by grabbing her wrist and not letting go.  Also he introduces himself by telepathy-dump, including “the two pieces of wood, the image of a man in pain,” and tells her it’s “the call of the First Forbidden One and the Second Forbidden One and the Third Forbidden One.” The Trinity, like you’ve never seen them (or it) before!  I guess.

Onward, past the Deep Dry Lake of the Damned Irene, resisting the temptation to lie down with the skeletons and die, to “the final source and the mystery, the Quel of the Thirteenth Nile,” where there are trees and caves, and fruits, melons, and grain growing, and evidence that other people used to live there, and also some surviving chickens running around.  Celalta declares, “We’re Adam and Eve in a way.  It’s not up to us to be given a god or to be given a faith.  It’s up to us to find the power, and this is the quietest and last of the searching places.” Et cetera.  Celalta says she’ll start the fire if O’Neill will go catch some chickens.

Well, this is pretty ridiculous.  It’s obviously some sort of religious allegory, reminding me a little of my ill-fated glancing encounter with The Pilgrim’s Progress, told in an often sonorous style but a plain vocabulary, like a negotiation between the King James Bible and Fun with Dick and Jane (that’s not a complaint).  But the point is a little elusive.  I get that at least one of the two is thinking about Adam and Eve, since she says it straight out.  But then what?  Mr. Smith owes us one more story in the series, catching up with Casher and Celalta and their inevitable children after ten years or so in isolation, living on feral chickens roasted in a cave.  But you know it won’t happen.

Two stars for this shaggy God epic.  As exasperating follies go, it’s at least readable and amusing.

The Comet Doom, by Edmond Hamilton

The reprints are an exceedingly mixed bag.  Surprisingly, the best is also the most archaic, Edmond Hamilton’s The Comet Doom, from the January 1928 Amazing.  There’s a big green comet passing by, and it turns out it’s inhabited by atomic-powered metal beings with tentacles who used to have organic bodies but gave them up.  These folks have about used up the comet’s resources and want to replenish their stores by carrying off a handy planet, ours to be precise.  In fact they have just yanked the Earth out of its orbit.  To further their scheme, they land on a lake island and snatch our heroes, Coburn and Hanley, and offer them metal bodies and immortality if they will help out in the liquidation of their species.

Hanley goes for it, Coburn escapes.  About this time, Marlin—the story’s narrator—is passing by the island in a boat which is half-destroyed by the comet, swims to shore, and encounters Coburn, who recruits him to the human cause.  They attack the cometeers and Coburn is killed, but the already-transplanted Hanley, in a final moment of human loyalty, destroys the machine that is steering Earth towards the comet, along with the comet-people present.  Doom is foiled.

This one is reasonably readable, mostly done in a style that reflects close attention to H.G. Wells, with echoes of both The Star and The War of the Worlds, despite the pulpish plot.  Two stars by today’s standards, probably a standout by those of its time.

Restricted Area, by Robert Sheckley

The other reprints are from the brief high-budget, and relatively high-brow, flowering of the Ziff-Davis magazines during 1952 and 1953, immediately after the magazine went from pulp size to digest size.  Robert Sheckley’s Restricted Area, from the June-July 1953 Amazing, is one of the slick but empty and cartoony pieces he produced in quantity at the beginning of his career, along with the more incisive ones. 


by Greisha Dotzenko

Space explorers land on a paradisical planet–wonderful climate, no germs, no rocks, lots of colorful friendly animals ready to hang out and play, and a giant steel shaft ascending to the clouds.  But after a while, the animals start to slow down and keel over.  Connect the dots.  Glib and facile, and the author knows it—this one hasn’t been in any of Sheckley’s multiple collections to date.  Two stars, barely.

Final Exam, by Chad Oliver


by Ashman

Final Exam by the sometimes redoubtable Chad Oliver, from the November/December 1952 Fantastic, is also from what we might call the Intermission, or Respite, between the Ziff-Davis magazines’ last gasp as pulps and their monotonous and purposely formulaic low-budget era of the mid- and late 1950s.  Like much of Oliver’s work, it reflects his anthropological bent (actually, a pretty straight-line bent—he’s become an anthropology professor at the University of Texas), but strikes an unusually sour note.  Professor La Farge’s class in Advanced Martian History is on a field trip to see and condescend to some of the colorful and primitive surviving Martians, but the time for the Martians to turn the tables has arrived in this heavy-handed satire.  Two stars, barely. 

Summing Up

Well, a couple more hours we’ll never get back, and not much to show for it, except an eccentric misfire from a sometimes brilliant writer, and a tolerable relic of a bygone era.  Next?



[Come join us at Portal 55, Galactic Journey's real-time lounge! Talk about your favorite SFF, chat with the Traveler and co., relax, sit a spell…]




[March 12, 1965] Sic Transit (April 1965 Amazing)


by John Boston

No-Sale, Sale

The big news, previously rumored, is that Amazing and its stablemate Fantastic are to change hands.  The April Science Fiction Times just arrived, with the big headline “ ‘AMAZING STORIES’ AND ‘FANTASTIC’ SOLD TO SOL COHEN.” Cohen is the publisher of Galaxy, If, and Worlds of Tomorrow, but will resign at the end of next month to take up his new occupation. 

Why is this happening?  Probably because circulation, which had been increasing, started to decline again in 1962 (when I started reviewing it!).  The SF Times article adds, tendentiously and questionably, that “the magazine showed what appeared to be a lack of interest by its editors.” Read their further comment and draw your own conclusions on that point.

Whatever the reason, it’s done.  The last Ziff-Davis issues of both magazines will be the June issues.  Whether they will continue monthly is not known, nor is who will edit the magazine—presumably meaning Cele Lalli is not continuing. 

The Issue at Hand


by Paula McLane

The previous two issues were much improved over their predecessors.  Of course the improvement could not be sustained (viz. the current issue), but it has at least reverted to the slightly less mean than on some occasions.

The Shores of Infinity, by Edmond Hamilton


by George Schelling

Who was it who first said “If you like this sort of thing, this is the sort of thing you will like”?  Dorothy Parker?  Mark Twain?  Pliny the Elder?  Anyway, Edmond Hamilton’s latest in his backwards-looking saga of the far future featuring the Star Kings may not even rise to the level of that truism.  The generically titled The Shores of Infinity spends about 10 of its 33 or so pages on background and build-up, and then the generically named John Gordon of ancient (i.e., contemporary) Earth takes off to the capital of the Galaxy to hobnob with the Emperor in the imperial city of Throon. 

There, he learns of more rumors of the previously encountered mind-controlling menace, is sent off on a dangerous secret mission to investigate, is captured, victimized by treachery and benefited by its reversal, and the author seems as bored by the whole perfunctory and hackneyed business as I was.  It’s ostentatiously inconclusive, so we are guaranteed more.  To paraphrase W.C. Fields, second prize would be two more sequels.

A recurring theme here is Gordon’s alienation from the beautiful Princess Lianna.  This seems to be the special Woman Trouble issue of Amazing; see below.  (Note to Betty Friedan: if the viewpoint characters were female, it would of course be the Man Trouble issue.  Tell women more of them should be writing SF.) Also worth mentioning is the cover, by Paula McLane, which portrays a giant space-helmeted visage peering into a room where two small diaphanous figures are trying to close the door against him.  Obviously metaphorical, right?  Actually, it’s a quite literal rendering of a passing scene in the story.

Anyway, this one has little going for it except the usual space-operatic rhetoric (“The vast mass of faintly glowing drift that was known as the Deneb Shoals, they skirted.  They plunged on and now they were passing through the space where, that other time, the space-fleets of the Empire and its allies had fought out their final Armageddon with the League of the Dark Worlds.”) The problem with this decoration is there’s not much here to decorate.  Two stars, probably too generous.

No Vinism Like Chau-Vinism, by John Jakes


by George Schelling

John Jakes’s long novelet No Vinism Like Chau-Vinism [sic], the title obviously a play on the all too familiar song There’s No Business Like Show Business, inhabits the territory demarcated by Pohl and Kornbluth’s The Space Merchants, Ron Goulart on a good day, and the Three Stooges.  In the future, everything is showbiz.  The populace is entertained and distracted by constant scripted and broadcast warfare between commercial forces, in this case the American Margarine Manufacturers Association versus the United Dairy Expedition Force, fighting of course in Wisconsin.

The script is disrupted by the appearance of a real gun in the hands of the rogue margarineer (Jakes calls them margies) Burton Tanzy of the Golden-Glo Margarine Company, who executes a cow on live TV, initiating a slapstick plot in which among other things besides real guns, the fake commercial war turns into a real union dispute.  The protagonist Gregory Rooke of the Dairy forces must try to calm things down and restore order, lest the public get wind of the fraudulence of their world.  His chief antagonist proves to be his ex-wife, who has dumped him to pursue her overweening ambitions, consistently with the Woman Trouble motif, though things work out better for Rooke than for poor forlorn John Gordon.

This story is actually quite amusing; Jakes has a knack for the farcical tall tale.  Unfortunately it goes on too long and palls a bit by the end, keeping the final reckoning down to three stars.

De Ruyter: Dreamer, by Arthur Porges

Arthur Porges contributes Ensign De Ruyter: Dreamer, another in his tiresome series about the space navy guy who triumphs over cartoonishly stupid extraterrestrials through the clever use of basic science—in this case, it’s Fun with Electromagnetism.  These stories barely rise to the level of filler.  One star.

Greendark in the Cairn, by Robert Rohrer

Robert Rohrer continues in his “almost there” vein with Greendark in the Cairn, featuring a space captain who is either going nuts, or is being driven nuts by transmissions from the extraterrestrial enemy, but figures out how to defeat them and prevent himself from giving the game away.  It’s gimmicky and facile but well rendered.  Two stars, but do try again.

Speech Is Silver, by John Brunner

The earnest John Brunner undertakes a satire of American commercialism in Speech Is Silver, in which the protagonist Hankin is selected in the Soundsleep company’s “Great Search” for the man with the perfect voice for a program of sleep counseling—that is, counseling while one sleeps on how to handle the difficulties of the day.  Except of course that’s not really how things work, and his new-found fame destroys his life.  (Also—Woman Trouble again—his wife, who prodded him to enter the Soundsleep competition, leaves him, apparently because he was never ambitious enough, like Rooke’s wife in No Vinism [etc.].) At the point where Soundsleep decides to replace him with a younger near-double, Hankin detonates.

Like the Jakes story, this one is an acid satire on contemporary image-mongering American capitalism.  Its problem, aside from being too long (also like Jakes), is that it isn’t crazy enough.  Brunner’s calm and methodical style and storytelling muffle the plot’s antic qualities.  Maybe he should get Jakes to introduce him to the Three Stooges.

Two stars.

Religion in Science Fiction: God, Space, and Faith, by Sam Moskowitz

Sam Moskowitz forges on with Religion in Science Fiction: God, Space, and Faith, rendered on the cover as Science-Fiction Views of God With a Profile of C.S. Lewis.  It’s a rambling description of various SF stories on religious themes, starting reasonably interestingly with several older books that most readers will not have heard of, continuing with the promised account of C.S. Lewis, followed by Stapledon, Heinlein, Wyndham, Simak, Leiber, Keller, Bradbury, Blish, Miller, del Rey, Boucher, and Farmer.

In this recitation Moskowitz manages to miss Arthur C. Clarke’s Hugo-winning The Star and his equally well-known The Nine Billion Names of God; Isaac Asimov’s The Last Question; Leigh Brackett’s tale of a different sort of theocracy, The Long Tomorrow; Harry Harrison’s anti-theological polemic The Streets of Ashkelon; Katherine MacLean’s acerbic Unhuman Sacrifice; and, astonishingly, Heinlein’s Stranger in a Strange Land.  Remarkably, he says, “Heinlein, who introduced [religion] into the science fiction magazines, has long since tired of it as a focal plot device.”

But this (mis-)observation does not deter him from his conclusion: “Neverthess, the decision is made.  The science fiction writer has come to the conclusion that scientific advance will not mean the end of belief.  He feels certain that a truly convincing portrayal of a hypothetical future cannot be made without considering the mystical aspects of man.”

Can we say fallacy of composition?  Two stars: another “almost,” some interesting material, some of it unfamiliar, the whole brought down by Moskowitz’s fatuous generalizations.

Summing Up

So, mostly not too bad (except for the egregious Ensign Ruyter), but mostly not as good as it should and could have been either.  There have been issues that would make one regret that this regime is ending, but this isn’t one of them.






[February 26, 1965] Dare to be Mediocre (February Galactoscope #2)

This second Galactoscope for February involves entries from both sides of the Atlantic.  It also introduces our newest writer, a most interesting Briton who we are most grateful to have; there's so much going on in the UK these days!

Dare by Philip Jose Farmer


By Jason Sacks

I’ve become a big fan of Philip Jose Famer over the years. Which is why I’m frustrated I didn’t enjoy his newest book, Dare, as much as I wanted to.

Like most of you, I became familiar with Farmer when I first read his famous short story “The Lovers.” I was captivated by Famer’s smart prose, his intriguing depiction of love between a human and alien, and most of all by his focus on human emotions while exploring thoroughly unique alien worlds.

Farmer has continued to build that reputation over the last decade, culminating in (at least to me) his imaginative world building with his outstanding 1964 novella “The Day of the Great Shout”,  which was set in his fantastical and intriguing Riverworld. With that story, it began to feel like Farmer was on the verge of taking his next leap forward as a writer, fulfilling the promise he showed during his first fecund period, during the 1950s, when he was nominated as Most Promising New Talent.

Unfortunately, Dare doesn’t quite demonstrate the virtuosity one expects from our current group of budding science fiction masters.

Oh, Dare has elements of uniqueness and sparks of something special. The world Farmer creates is broad and diverse, with clues dropped of the same sorts of cosmic chessmasters who might have created the Riverworld.

The planet Dare is a fantastical place, part utopia and part dystopia, inhabited by a heady and fascinating mix of humans and fantasy-like creatures.

In one of the most interesting twists (which doesn’t pay off in the book) the humans on this planet are the members of the original Jamestown settlement on Earth, who landed in Virginia at the dawn of the era of colonization and then disappeared before the next boatload of Brits landed in America. Farmer answers the lingering mystery of their disappearance in the most science-fiction way possible: these settlers have been kidnapped to that aforementioned alien world, in which they find themselves attempting to survive and continue the way of life for which they left England in the first place.

As happens with every generation, where parents set rules, the children will defy the rules. Love will find its way, even if the love is between two different species.

The other inhabitants of the planet are a curious mix of creatures which seem to emerge from Terran mythology. There are mandrakes and talking dragons and annoying unicorns – a clever running gag of the book paints unicorns as stupid, emotional animals and far from childrens’ fantasies – and a group of satyr type creatures. Naturally the satyrs cavort about in the nude and naturally the human boy falls in love with a female of the species named R’li.

A triple novel?

Dare really reads like three books – or maybe three short stories – under one cover.

The first third of the book dwells mainly on the romance between Jack and R’li. This section is sweet and a bit sexy and reminds me of a variant on “The Lovers”. Much of the middle third of the book shows the humans’ fury at the boy’s indiscretion, and is full of action and intrigue. However, the charm of the first third is tossed away for more of a violent, action-adventure story, and the transition between those two sections happens awkwardly, making the book feel like it’s arguing with itself. In the last third, Farmer takes the plot into more of a science fiction battle territory as a ship arrives and changes everything on the planet.

None of these storylines cohere well with the others. There’s a feeling that Farmer wrote three short stories set in this world and then just grafted them together, never mind that the tone shifts wildly and the book doesn't effectively build to a satisfying conclusion. A reader finishes this book a bit stunned, unsure what to make of the mysterious mélange Farmer has delivered.

More than that, there’s just so much here that feels underdeveloped. I wanted to learn more about the dislocation the Virginians felt, to understand more about the alien society, and to understand what force brought all these creatures to the planet. Unusually for Farmer, this book felt more about the surface and less about the depth, making for a jarring and ultimately frustrating read.

I still hope for good things from Farmer, but Dare represents a step backwards on the road to mastery.

Rating: 2 stars


New Writings in SF 3


by Mx. Kris Vyas-Myall

If you've been a science fiction fan in Britain anytime in the last decade you likely know John Carnell. He was an editor on Britain’s first fanzine, New Worlds, before the war and revived it as a professional fiction venue in the '40s. He then further expanded to Science Fantasy and Science Fiction Adventures, becoming rather like Britain’s version of Frederik Pohl.

However, with the latter shutting down last year and declining sales on the other two titles, Carnell decided to take another leaf from Pohl’s book to move away from publishing magazines and become a literary agent, and to try his hand at publishing original anthologies.

His stated aims in the first publication were as follows:

  • Only publish either original stories or those not likely seen by the vast majority of readers
  • Introduce new short fiction to the general public rather than just science fiction afficionados
  • Introduce new styles, ideas and writers to the genre

I can definitely say he has been successful in the first one and probably in the second, but I am not so sure on the final point.

Firstly, most of the writers had already been writing for New Worlds; the only truly new ones so far have been John Rankine & G. L. Lack. And I would not say this work is that experimental — rather it has been solid in established fields. Ironically these aims seem like they might be being better achieved by Moorcock and Bonfiglioli in their new management of Carnell's old magazines.

What we have had in the previous two issues of New Writings are solid stories of the type we would expect from these writers, even those like Brian Aldiss (whose work I always love). I would never place these works among their top range, but even the lower tier authors brought over from Carnell’s last years on New Worlds are still producing readable work for the first two volumes as well as #3, which I shall now discuss:

The Subways of Tazoo, by Colin Kapp

In our first story we follow an archeological dig as they attempt to uncover an extinct civilization on a hostile world. The story is largely told through rather unnatural conversation, but the way it unfolds and gives us more information about the Tazoon is rather interesting. A low three stars.

The Fiend, by Frederik Pohl

Speaking of Pohl, his influence appears again with this reprint from Playboy (described wonderfully by Carnell as “an American magazine devoted to the broadest of broad policies of masculine appeal”). Here Pohl attempts a dark tale of an interstellar voyage captain’s obsession with a frozen passenger, but comes across as creepy in the wrong way. Two stars for me but one that may appeal to other new wavers.

Manipulation, by John Kingston

The first of two stories by regular Science Fantasy contributor Keith Roberts (under, what I believe is, a new pseudonym) where he gives us a stylish and evocative tale of a man dealing with having psychic powers. This fresh take, whilst not as highly experimental as is being published by Moorcock, represents the closest to the fulfillment of Carnell’s stated aims. Four stars.

Testament, by John Baxter

The return of another New Worlds regular with this vignette on survival in a dying world. These kinds of apocalypses are very much in vogue right now but Baxter manages a deft and memorable work. Four stars.

Night Watch, by James Inglis

A second very short piece in a row. This one treads over some well-worn ground but does it well. A solid three stars.

Boulter's Canaries, by Keith Roberts

In his second story for the anthology Roberts asks, is there a scientific explanation for ghosts? The resulting answer is less satisfactory than other recent attempts. Two stars.

Emreth, by Dan Morgan

This is a story from an old hand returning to SF writing after a four-year hiatus. It has incredibly strong and memorable moments but doesn't tie well enough together for me to get beyond three stars.

Space Master, by James H. Schmitz

Schmitz, as a longstanding and prolific American author, seems like an odd fit to finish out this collection. If you like the kind of work he does you may enjoy this story, I personally do not. Two stars.

In Conclusion

So overall this is pretty much down the middle. None of the stories within seem destined to be all-time classics but none are truly awful; even those I disliked I can see they may well appeal to others. Solid and competent work.

By all accounts these collections have been pretty well received by the science fiction buying public over here, and along with increasing sales on New Worlds and Science Fantasy, it seems like British Science Fiction is in safe hands.


Like Watching a Movie


by Gideon Marcus

Another month, another Ace Double.  This one is designated M-111, and like most of the rest of the books in the series, it offers two mildly interesting adventure stories.  In this case, I felt the writing exceptionally vivid; both books would make good film adaptations, I think.

Fugitive of the Stars, by Edmond Hamilton


by Jack Gaughan

Horne, 1st Navigator on the Vega Queen, makes landfall on the Fringe planet of Skereth.  Skereth is on the verge of accepting an invitation to join the galactic Federation of planets, and they are sending the envoy, Morivenn, to effect the union.  In a back alley on Skereth's capital, Horne and his 2nd Navigator are beset by hoodlums, and the latter crewman is rendered unable to work.  Luckily, an eager-beaver Skerethian named Ardric is a qualified 2nd Navigator.

He's also an anti-Federation agent, and he manages to destroy the Vega Queen, killing most of its passengers and crew before getting away.  Horne is courtmartialed for negligence, but he flees justice before he can be sentenced.  Now on the hunt for Ardric, his goal is to clear his name — and discover what secret makes Skereth is so hell-bent on staying out of the Federation.

If this plot sounds familiar, it may be because you read the novella on which it's based (basically the latter two thirds of the book) came out as Fugitive of the Stars in one of the last issues of Imagination more than seven years ago. 


by Malcolm Smith

Thus, there's no way the title is meant to evoke the current TV show staring David Janssen (The Fugitive, natch).  In any event, Edmond Hamilton (Mr. Leigh Brackett) does a fine job with this riproaring space opera, and the expansion into a full-length novel only improves the story.  The best exchange in the book is this one, while Horne and Morivenn's daughter, Yso, are dogfighting Ardris' goons in hover cones:

Yso: "What's the matter?  Haven't you ever seen a woman fight before?"

Horne: "When I was in the Navy, some of my best men were women.  Are you Navy?"

Yso: "Skereth Planetary."

Three and a half stars.

Land Beyond the Map, by Kenneth Bulmer


by Jerome Podwil

Rollie Crane, a listless dilettante millionaire, had a traumatic experience as a child.  On a road trip through Ireland, his father, using a strange half-map, drove his family's car into a strange alternate dimension.  Therein, the ground heaved with chaos, clanking treaded things chased them, and strange towers bisected the horizon.  All of this lay half-forgotten until the stormy night that Polly Gould arrived at Crane's mansion with stories about a similar map, which had swallowed her former boyfriend and his new love many years prior.

The two decide to return to Ireland and search every antique bookshop until they find the map.  But what will they find when they reach the uncharted zone?  And who is this sinister McArdle character who shows up to warn them off their task?  Worst yet, what are these floating baleful eyes that burn with golden fire and vaporize at a glance?

I have to say that prolific British author Kenneth Bulmer had never really impressed me to date.  Land, on the other hand, is a fun romp.  In many ways, it feels like an Edgar Rice Burroughs story, with little reliance on technology, captivating scenery, and two strong characters who clearly fancy each other but can't confess their feelings until the very end.

Where the tale falls down is the conclusion, in which Crane has no real role.  He watches lots of exciting things happen, but he affects them not at all.  It's a shame and something of a cheat; surely Bulmer could have given Crane and/or Gould something to do at the climax.

So, three and a half stars for a pleasant time whose imagery will stay with you even if the plot doesn't.

(by the way, I've now learned that this story is also a reprint of sorts, an expansion of Map Country from the February 1961 Science-Fantasy.  It seems largely the same — just fuller.)


by Brian Lewis


That's it for February!  March promises to be a light month for books — good thing since we've been flooded with magazines!  Stay tuned…






[September 22, 1964] Fall back!  (October 1964 Fantasy and Science Fiction)

[Don't miss your chance to get your copy of Rediscovery: Science Fiction by Women (1958-1963), some of the best science fiction of the Silver Age.  If you like the Journey, you'll love this book (and you'll be helping us out, too!)



by Gideon Marcus

To every thing there is a season

Even in timeless southern California, we have seasons.  In the Imperial Valley, it is joked, there are four: Hot, Bug, Stink, and Wind.  Here in San Diego, spring comes in summer, summer comes in fall, fall comes in winter, and winter not at all.

Yet here and there, we see a deciduous tree start to change color.  The end-of-summer mornings have a hint of chill in them.  Things proceed in an endless cycle.

The same is true of The Magazine of Fantasy and Science FictionLast month, I raved over a superlative issue, an increasing rarity under the current editorship of one Avram Davidson.  I am sad to report that things are back to form in this month's issue.

I think part of the problem is that, as Davidson cheerfully confesses, he's not really into science fiction.  He bounces off the truly hard stuff, like Martin Caidin's quite good Countdown and fills his magazine with fantastic fluff…and then has the temerity to complain that people don't sent him plain old rocket stories anymore!

On the other hand, the rumor has been confirmed — Davidson has moved to Berkeley from Mexico, and someone else is taking over the magazine.  I hear that Joe Ferman, currently publisher, will take the helm, but that his son, Ed, will do all the work.  I look forward to seeing what they bring to the table.

But first, let's take a look at what is possibly Davidson's last editorial effort, what he optimistically calls an "All Star Issue".

Autumn Harvest


by Chesley Bonstell

Once again, the cover is stunning — and utterly unrelated to the contents of the issue.  It's a depiction of an ion-drive propelled ship off of Mars, and it's from the book Beyond the Solar System, presumably available on bookshelves near you.

Purple Priestess of the Mad Moon, by Leigh Brackett

The first of the All Stars is the legendary Leigh Brackett, queen of pulp and accomplished screenwriter.  This tale actually began as a joke nine years ago, when a fictional title was created to represent the kind of fiction Brackett excelled at.  Purple Priestess is the author's attempt to turn a joke into reality.

It has all the hallmarks of a pulp Mars, from the thin air to the drying canals, the ancient natives who speak High and Low Martian.  And, of course, out in the frigid deserts lies an antediluvian evil so terrible that none can experience its presence and fail to gibber.

I enjoyed Lovecraft's stories well enough in the '30s , but I'm disappointed to find one presented unironically in what was once the premier SF mag.  Two stars.

The Pro, by Edmond Hamilton

The subsequent piece, by Brackett's husband (of similar vintage) is better.  One can't help but see a bit of the autobiographical in this story about a science fiction author who finally gets to see the rocketships he created in fiction become reality at the Cape.  Only the launching of the latest of them is not a joyous occasion, for the writer's child is the pilot.  Even if the mission goes well, it marks a final rift between father and son, one the writer is sure can never be bridged.

A bit maudlin but enjoyable.  Three stars.

Stomata, by Theodore L. Thomas

Thomas' latest short story idea disguised as a non-fiction article takes the idea of stomata, the pores that allow plants to respire, and posits an race that uses them for everything — breathing, eating, excreting.

I don't know how plausible the idea is.  On the other hand, Pinky the Blob, debuting in one of my upcoming books, employs exactly this mechanism.  Great minds think alike.

Three stars.

Maid to Measure, by Damon Knight

Five years ago, Damon Knight came out with What Rough Beast, a story so excellent that I'm reading it again in the Spanish edition of F&SF

Maid to Measure, a joke-ending vignette about a shape-changing girl, is as trivial as Beast is momentous.

Two stars.

Little Anton, by Reginald Bretnor

Bretnor is perhaps better known to the readers of F&SF as Grendall Briarton, composer of the recently finished series of "Feghoot" pun stories.  After reading this awful reprint, the story of an idiot savant inventor with a tedious Swiss accent and a penchant for pinching posteriors, I'm actually nostalgic for Briarton.

One star.

First and Rearmost, by Isaac Asimov

Doc A. turns in an above average science article this month, all about how gravity stacks up to the other three primal forces of the universe: electromagnetism, the weak nuclear force, and the strong nuclear force (his omission of love and money are probably deliberate).  It's all stuff I knew already, but he lays it out nicely for laymen.

Four stars.

The Year of the Earthman, by Hogan Smith

An old, radiation-scarred astronaut goes AWOL to marry a lovely extraterrestrial lass, dying just moments after he learns that they will have a son.  And then we learn the truth of the space traveler's existence.

Not a bad tale, though it makes little scientific sense.  Also, Hogan Smith is the opposite of an All Star — this is is first story!  But he's from San Diego, so all is forgiven.

Three stars.

In What Cavern of the Deep, by Robert F. Young

Robert F. Young's little autobiography at the front of Cavern is quite interesting.  Like me, he came into the genre by way of Burroughs and then Wells, and also like me, he tried making an honest living before deciding that writing is the most fun one can have with their hands — especially if one gets paid for it!

Young writes stories inspired by mythology and folklore, and while he has come out with some of my very favorite stories, his works from the last several years have been disappointing and mawkish.  His latest falls somewhere inbetween.

David Stuart is a poor young man made rich through inheritance from an uncle.  While investigating the deceased's estate, he comes across two swimming sisters and promptly falls in love with Helen, the blonder of them.  But the ensuing marital bliss is dashed by the revelation that Helen is growing taller by the week, approaching titanic proportions after just a year.  It's sort of an inverse of Richard Matheson's The Incredible Shrinking Man.  At the same time, David's wife becomes more and more enamored with bodies of water, swimming constantly and even growing gill slits.

Is Helen a beast of the sea?  An alien?  And is the story going to end horrifically (as set up in the prologue) with David hurtling five smooth stones to smite his monstrous love?

Cavern is a bit of a departure from Young's previous stories in that, though he makes conscious references to the biblical King David, this is more to obscure the plot than to outline it.  The piece is told with Young's usual excellent facility, and I found myself eager to get to the end.

On the other hand, the end is just a bit too pat, too clearly presented to be very satisfying.  What could have been a 4 or even 5 star story ends up on the high end of 3.

Empty Cornucopia

If this be Davidson's swansong, he picked a sad note to go out on.  Maybe he's got one issue more in him before he shuffles off F&SF's bridge — I'd like to have fonder memories of this phase of his career!


[Come join us at Portal 55, Galactic Journey's real-time lounge! Talk about your favorite SFF, chat with the Traveler and co., relax, sit a spell…]